Уравнения тригонометрии формулы: Методы решения тригонометрических уравнений. Тригонометрические уравнения — формулы, решения, примеры Разложение на множители тригонометрических уравнений примеры

Содержание

Методы решения тригонометрических уравнений. Тригонометрические уравнения — формулы, решения, примеры Разложение на множители тригонометрических уравнений примеры

Методы решения тригонометрических уравнений.

Решение тригонометрического уравнения состоит из двух этапов: преобразование уравнения для получения его простейшего вида (см. выше ) и решение полученного простейшего тригонометрического уравнения. Существует семь основных методов решения тригонометрических уравнений.

1. Алгебраический метод.

(метод замены переменной и подстановки).

2. Разложение на множители.

П р и м е р 1. Решить уравнение: sin x + cos x = 1 .

Р е ш е н и е. Перенесём все члены уравнения влево:

Sin x + cos x – 1 = 0 ,

Преобразуем и разложим на множители выражение в

Левой части уравнения:

П р и м е р 2. Решить уравнение: cos 2 x + sin x · cos x = 1.

Р е ш е н и е. cos 2 x + sin

x · cos x – sin 2 x – cos 2 x = 0 ,

Sin x · cos x – sin 2 x = 0 ,

Sin x · (cos x – sin x ) = 0 ,

П р и м е р 3. Решить уравнение: cos 2 x – cos 8 x + cos 6 x = 1.

Р е ш е н и е. cos 2 x + cos 6 x = 1 + cos 8 x ,

2 cos 4x cos 2x = 2 cos ² 4x ,

Cos 4x · (cos 2x – cos 4x ) = 0 ,

Cos 4x · 2 sin 3x · sin x = 0 ,

1). cos 4x = 0 , 2). sin 3x = 0 , 3). sin x = 0 ,

3. Приведение к однородному уравнению.

Уравнение называется однородным от носительно sin и cos , если все его члены одной и той же степени относительно sin и cos одного и того же угла . Чтобы решить однородное уравнение, надо:

а ) перенести все его члены в левую часть;

б ) вынести все общие множители за скобки;

в ) приравнять все множители и скобки нулю;

г ) скобки, приравненные нулю, дают однородное уравнение меньшей степени, которое следует разделить на

cos (или sin ) в старшей степени;

д ) решить полученное алгебраическое уравнение относительно tan .

sin 2 x + 4 sin x · cos x + 5 cos 2 x = 2.

Р е ш е н и е. 3sin 2 x + 4 sin x · cos x + 5 cos 2 x = 2sin 2 x + 2cos 2 x ,

Sin 2 x + 4 sin x · cos x + 3 cos 2 x = 0 ,

Tan 2 x + 4 tan x + 3 = 0 , отсюда y 2 + 4y +3 = 0 ,

Корни этого уравнения: y 1 = — 1, y 2 = — 3, отсюда

1) tan

x = –1, 2) tan x = –3,

4.

Переход к половинному углу.

Рассмотрим этот метод на примере:

П р и м е р. Решить уравнение: 3 sin x – 5 cos x = 7.

Р е ш е н и е. 6 sin (x / 2) · cos (x / 2) – 5 cos ² (x / 2) + 5 sin ² (x / 2) =

7 sin ² (x / 2) + 7 cos ² (x / 2) ,

2 sin ² (x / 2) – 6 sin (x / 2) · cos (x / 2) + 12 cos ² (x / 2) = 0 ,

tan ² (x / 2) – 3 tan (x / 2) + 6 = 0 ,

. . . . . . . . . .

5. Введение вспомогательного угла.

Рассмотрим уравнение вида :

a sin x + b cos x = c ,

Где a , b , c – коэффициенты; x – неизвестное.

Теперь коэффициенты уравнения обладают свойствами синуса и косинуса , а именно : модуль ( абсолютное значение ) каждого из них не больше 1, а сумма их квадратов равна 1 . Тогда можно обозначить их соответственно как cos и sin (здесь — так называемый вспомогательный угол ), и наше уравнение прини

Основными методами решения тригонометрических уравнений являются: сведение уравнений к простейшим (с использованием тригонометрических формул), введение новых переменных, разложение на множители.

Рассмотрим их применение на примерах. Обратите внимание на оформление записи решений тригонометрических уравнений.

Необходимым условием успешного решения тригонометрических уравнений является знание тригонометрических формул (тема 13 работы 6).

Примеры.

1. Уравнения, сводящиеся к простейшим.

1) Решить уравнение

Решение:

Ответ:

2) Найти корни уравнения

(sinx + cosx) 2 = 1 – sinxcosx, принадлежащие отрезку .

Решение:

Ответ:

2. Уравнения, сводящиеся к квадратным.

1) Решить уравнение 2 sin 2 x – cosx –1 = 0.

Решение: Используя формулу sin 2 x = 1 – cos 2 x, получаем

Ответ:

2) Решить уравнение cos 2x = 1 + 4 cosx.

Решение: Используя формулу cos 2x = 2 cos 2 x – 1, получаем

Ответ:

3) Решить уравнение tgx – 2ctgx + 1 = 0

Решение:

Ответ:

3. Однородные уравнения

1) Решить уравнение 2sinx – 3cosx = 0

Решение: Пусть cosx = 0, тогда 2sinx = 0 и sinx = 0 – противоречие с тем, что sin 2 x + cos 2 x = 1. Значит cosx ≠ 0 и можно поделить уравнение на cosx. Получим

Ответ:

2) Решить уравнение 1 + 7 cos 2 x = 3 sin 2x

Решение:

Используем формулы 1 = sin 2 x + cos 2 x и sin 2x = 2 sinxcosx, получим

sin 2 x + cos 2 x + 7cos 2 x = 6sinxcosx
sin 2 x – 6sinxcosx+ 8cos 2 x = 0

Пусть cosx = 0, тогда sin 2 x = 0 и sinx = 0 – противоречие с тем, что sin 2 x + cos 2 x = 1.
Значит cosx ≠ 0 и можно поделить уравнение на cos 2 x. Получим

tg 2 x – 6 tgx + 8 = 0
Обозначим tgx = y
y 2 – 6 y + 8 = 0
y 1 = 4; y 2 = 2
а) tgx = 4, x= arctg4 + 2 k , k
б) tgx = 2, x= arctg2 + 2 k , k .

Ответ: arctg4 + 2 k , arctg2 + 2 k, k

4. Уравнения вида a

sinx + b cosx = с, с ≠ 0.

1) Решить уравнение .

Решение:

Ответ:

5. Уравнения, решаемые разложением на множители.

1) Решить уравнение sin2x – sinx = 0. n arcsin a + \pi n, n \in Z`

2. Уравнение `cos x=a`

При `|a|>1` — как и в случае с синусом, решений среди действительных чисел не имеет.

При `|a| \leq 1` имеет бесконечное множество решений.

Формула корней: `x=\pm arccos a + 2\pi n, n \in Z`

Частные случаи для синуса и косинуса в графиках.

3. Уравнение `tg x=a`

Имеет бесконечное множество решений при любых значениях `a`.

Формула корней: `x=arctg a + \pi n, n \in Z`

4. Уравнение `ctg x=a`

Также имеет бесконечное множество решений при любых значениях `a`.

Формула корней: `x=arcctg a + \pi n, n \in Z`

Формулы корней тригонометрических уравнений в таблице

Для синуса:
Для косинуса:
Для тангенса и котангенса:
Формулы решения уравнений, содержащих обратные тригонометрические функции:

Методы решения тригонометрических уравнений

Решение любого тригонометрического уравнения состоит из двух этапов:

  • с помощью преобразовать его до простейшего;
  • решить полученное простейшее уравнение, используя выше написанные формулы корней и таблицы.
    2 x=0`, `sin x(1-sin x)=0`. Тогда `sin x=0` или `1-sin x=0`.

    1. `sin x=0`, `x=\pi n`, `n \in Z`
    2. `1-sin x=0`, `sin x=-1`, `x=\pi /2+2\pi n, n \in Z`.

    Учитывая, что ` x \ne \pi+2\pi n, n \in Z`, решениями будут `x=2\pi n, n \in Z` и `x=\pi /2+2\pi n`, `n \in Z`.

    Ответ. `x=2\pi n`, `n \in Z`, `x=\pi /2+2\pi n`, `n \in Z`.

    Тригонометрия, и тригонометрические уравнения в частности, применяются почти во всех сферах геометрии, физики, инженерии. Начинается изучение в 10 классе, обязательно присутствуют задания на ЕГЭ, поэтому постарайтесь запомнить все формулы тригонометрических уравнений — они вам точно пригодятся!

    Впрочем, даже запоминать их не нужно, главное понять суть, и уметь вывести. Это не так и сложно, как кажется. Убедитесь сами, просмотрев видео.

    Тема: «Методы решения тригонометрических уравнений».

    Цели урока:

    образовательные:

    Сформировать навыки различать виды тригонометрических уравнений;

    Углубление понимания методов решения тригонометрических уравнений;

    воспитательные:

    Воспитание познавательного интереса к учебному процессу;

    Формирование умения анализировать поставленную задачу;

    развивающие:

    Формировать навык проводить анализ ситуации с последующим выбором наиболее рационального выхода из нее.

    Оборудование: плакат с основными тригонометрическими формулами, компьютер, проектор, экран.

    Начнем урок с повторения основного приема решения любого уравнения: сведение его к стандартному виду. Путем преобразований линейные уравнения сводят к виду ах = в, квадратные – к виду ax 2 + bx + c =0. В случае тригонометрических уравнений необходимо свести их к простейшим, вида: sinx = a , cosx = a , tgx = a , которые легко можно решить.

    В первую очередь, конечно, для этого необходимо использовать основные тригонометрические формулы, которые представлены на плакате: формулы сложения, формулы двойного угла, понижения кратности уравнения. Мы уже умеем решать такие уравнения. Повторим некоторые из них:

    Вместе с тем существуют уравнения, решение которых требует знаний некоторых специальных приемов.

    Темой нашего урока является рассмотрение этих приемов и систематизация методов решения тригонометрических уравнений.

    Методы решения тригонометрических уравнений.

    1. Преобразование к квадратному уравнению относительно какой-либо тригонометрической функции с последующей заменой переменной.

    Рассмотрим каждый из перечисленных методов на примерах, но более подробно остановимся на двух последних, так как два первых мы уже использовали при решении уравнений.

    1. Преобразование к квадратному уравнению относительно какой-либо тригонометрической функции.

    2. Решение уравнений методом разложения на множители.

    3. Решение однородных уравнений.

    Однородными уравнениями первой и второй степени называются уравнения вида:

    соответственно (а ≠ 0, b ≠ 0, с ≠ 0).

    При решении однородных уравнений почленно делят обе части уравнения на cosx для (1) уравнения и на cos 2 x для (2). Такое деление возможно, так как sinx и cosx не равны нулю одновременно – они обращаются в нуль в разных точках. Рассмотрим примеры решения однородных уравнений первой и второй степени.

    Запомним это уравнение: при рассмотрении следующего метода – введение вспомогательного аргумента, решим его другим способом.


    4. Введение вспомогательного аргумента.

    Рассмотрим уже решенное предыдущим методом уравнение:

    Как видим, получается тот же результат.

    Рассмотрим еще один пример:

    В рассмотренных примерах было, в общем, понятно, на что требуется разделить исходное уравнение, чтобы ввести вспомогательный аргумент. Но может случиться, что не очевидно, какой делитель выбрать. Для этого существует специальная методика, которую мы сейчас и рассмотрим в общем виде. Пусть дано уравнение.

    Формула решения косинусов частные случаи. Тригонометрические уравнения. Разложение на множители

    Простейшие тригонометрические уравнения решаются, как правило, по формулам. Напомню, что простейшими называются вот такие тригонометрические уравнения:

    sinx = а

    cosx = а

    tgx = а

    ctgx = а

    х — угол, который нужно найти,
    а — любое число.

    А вот и формулы, с помощью которых можно сразу записать решения этих простейших уравнений.

    Для синуса:

    Для косинуса:

    х = ± arccos a + 2π n, n ∈ Z

    Для тангенса:

    х = arctg a + π n, n ∈ Z

    Для котангенса:

    х = arcctg a + π n, n ∈ Z

    Собственно, это и есть теоретическая часть решения простейших тригонометрических уравнений. Причём, вся!) Совсем ничего. Однако, количество ошибок по этой теме просто зашкаливает. Особенно, при незначительном отклонении примера от шаблона. Почему?

    Да потому, что масса народу записывает эти буковки, не понимая их смысла совершенно! С опаской записывает, как бы чего не вышло…) С этим надо разобраться. Тригонометрия для людей, или люди для тригонометрии, в конце концов!?)

    Разберёмся?

    Один угол у нас будет равен arccos a, второй: -arccos a.

    И так будет получаться всегда. При любом а.

    Если не верите, наведите курсор мышки на картинку, или коснитесь рисунка на планшете. ) Я изменил число а на какое-то отрицательное. Всё равно, один угол у нас получился arccos a, второй: -arccos a.

    Следовательно, ответ можно всегда записать в виде двух серий корней:

    х 1 = arccos a + 2π n, n ∈ Z

    х 2 = — arccos a + 2π n, n ∈ Z

    Объединяем эти две серии в одну:

    х= ± arccos а + 2π n, n ∈ Z

    И все дела. Получили общую формулу для решения простейшего тригонометрического уравнения с косинусом.

    Если вы понимаете, что это не какая-то сверхнаучная мудрость, а просто сокращённая запись двух серий ответов, вам и задания «С» будут по плечу. С неравенствами, с отбором корней из заданного интервала… Там ответ с плюсом/минусом не катит. А если отнестись к ответу делово, да разбить его на два отдельных ответа, всё и решается.) Собственно, для этого и разбираемся. Что, как и откуда.

    В простейшем тригонометрическом уравнении

    sinx = а

    тоже получается две серии корней. Всегда. И эти две серии тоже можно записать одной строчкой. Только эта строчка похитрее будет:

    х = (-1) n arcsin a + π n, n ∈ Z

    Но суть остаётся прежней. Математики просто сконструировали формулу, чтобы вместо двух записей серий корней, сделать одну. И всё!

    Проверим математиков? А то мало ли…)

    В предыдущем уроке подробно разобрано решение (безо всяких формул) тригонометрического уравнения с синусом:

    В ответе получились две серии корней:

    х 1 = π /6 + 2π n, n ∈ Z

    х 2 = 5π /6 + 2π n, n ∈ Z

    Если мы будем решать это же уравнение по формуле, получим ответ:

    х = (-1) n arcsin 0,5 + π n, n ∈ Z

    Вообще-то, это недоделанный ответ.) Ученик обязан знать, что arcsin 0,5 = π /6. Полноценный ответ будет:

    х = (-1) n π /6 + π n, n ∈ Z

    Тут возникает интересный вопрос. Ответ через х 1 ; х 2 (это правильный ответ!) и через одинокий х (и это правильный ответ!) — одно и то же, или нет? Сейчас узнаем. )

    Подставляем в ответ с х 1 значения n =0; 1; 2; и т.д., считаем, получаем серию корней:

    х 1 = π/6; 13π/6; 25π/6 и так далее.

    При такой же подстановке в ответ с х 2 , получаем:

    х 2 = 5π/6; 17π/6; 29π/6 и так далее.

    А теперь подставляем значения n (0; 1; 2; 3; 4…) в общую формулу для одинокого х . Т.е возводим минус один в нулевую степень, затем в первую, вторую, и т.д. Ну и, разумеется, во второе слагаемое подставляем 0; 1; 2 3; 4 и т.д. И считаем. Получаем серию:

    х = π/6; 5π/6; 13π/6; 17π/6; 25π/6 и так далее.

    Вот всё и видно.) Общая формула выдаёт нам точно такие же результаты, что и два ответа по отдельности. Только все сразу, по порядочку. Не обманули математики.)

    Формулы для решения тригонометрических уравнений с тангенсом и котангенсом тоже можно проверить. Но не будем.) Они и так простенькие.

    Я расписал всю эту подстановку и проверку специально. Здесь важно понять одну простую вещь: формулы для решения элементарных тригонометрических уравнений есть, всего лишь, краткая запись ответов. Для этой краткости пришлось вставить плюс/минус в решение для косинуса и (-1) n в решение для синуса.

    Эти вставки никак не мешают в заданиях, где нужно просто записать ответ элементарного уравнения. Но если надо решать неравенство, или далее нужно что-то делать с ответом: отбирать корни на интервале, проверять на ОДЗ и т.п, эти вставочки могут запросто выбить человека из колеи.

    И что делать? Да либо расписать ответ через две серии, либо решать уравнение/неравенство по тригонометрическому кругу. Тогда исчезают эти вставочки и жизнь становится легче.)

    Можно подвести итоги.

    Для решения простейших тригонометрических уравнений существуют готовые формулы ответов. Четыре штуки. Они хороши для мгновенной записи решения уравнения. Например, надо решить уравнения:

    sinx = 0,3

    Легко: х = (-1) n arcsin 0,3 + π n, n ∈ Z

    cosx = 0,2

    Без проблем: х = ± arccos 0,2 + 2π n, n ∈ Z

    tgx = 1,2

    Запросто: х = arctg 1,2 + π n, n ∈ Z

    ctgx = 3,7

    Одной левой: x= arcctg3,7 + π n, n ∈ Z

    cos x = 1,8

    Если вы, блистая знаниями, мгновенно пишете ответ:

    х= ± arccos 1,8 + 2π n, n ∈ Z

    то блистаете вы уже, это. .. того… из лужи.) Правильный ответ: решений нет. Не понимаете, почему? Прочитайте, что такое арккосинус. Кроме того, если в правой части исходного уравнения стоят табличные значения синуса, косинуса, тангенса, котангенса, — 1; 0; √3; 1/2; √3/2 и т.п. — ответ через арки будет недоделанным. Арки нужно обязательно перевести в радианы.

    А если уж вам попалось неравенство, типа

    то ответ в виде:

    х πn, n ∈ Z

    есть редкая ахинея, да…) Тут надо по тригонометрическому кругу решать. Чем мы и займёмся в соответствующей теме.

    Для тех, кто героически дочитал до этих строк. Я просто не могу не оценить ваши титанические усилия. Вам бонус.)

    Бонус:

    При записи формул в тревожной боевой обстановке, даже закалённые учёбой ботаны частенько путаются, где πn, а где 2π n. Вот вам простой приёмчик. Во всех формулах стоит πn. Кроме единственной формулы с арккосинусом. Там стоит 2πn. Два пиэн. Ключевое слово — два. В этой же единственной формуле стоят два знака в начале. Плюс и минус. И там, и там — два.

    Так что, если вы написали два знака перед арккосинусом, легче вспомнить, что в конце будет два пиэн. А ещё наоборот бывает. Пропустит человек знак ± , доберётся до конца, напишет правильно два пиэн, да и спохватится. Впереди-то два знака! Вернётся человек к началу, да ошибку-то и исправит! Вот так.)

    Если Вам нравится этот сайт…

    Кстати, у меня есть ещё парочка интересных сайтов для Вас.)

    Можно потренироваться в решении примеров и узнать свой уровень. Тестирование с мгновенной проверкой. Учимся — с интересом!)

    можно познакомиться с функциями и производными.

    Простейшими тригонометрическими уравнениями называют уравнения

    Cos (x) = a, sin (x) = a, tg (x) = a, ctg (x) =a

    Уравнение cos (x) = a

    Объяснение и обоснование

    1. Корни уравнения cosx = а. При | a | > 1 уравнение не имеет корней, по-скольку | cosx | 1 или при а

    Пусть | а |

    у = cos х. На промежутке функция y = cos x убы-вает от 1 до -1. Но убывающая функция принимает каждое свое значение только в одной точке ее области определения, поэтому уравнение cos x = а имеет на этом промежутке только один корень, который по опреде-лению арккосинуса равен: x 1 = arccos а (и для этого корня cos x = а).

    Косинус — четная функция, поэтому на промежутке [-п; 0] уравнение cos x = а также имеет только один корень — число, противоположное x 1 , то есть

    x 2 = -arccos а.

    Таким образом, на промежутке [-п; п] (длиной 2п) уравнение cos x = а при | а |

    Функция y = cos x периодическая с периодом 2п, поэтому все остальные корни отличаются от найденных на 2пп (n € Z). Получаем следующую фор-мулу корней уравнения cos x = а при

    x = ±arccos а + 2пп, n £ Z.

    1. Частные случаи решения уравнения cosx = а.

    Полезно помнить специальные записи корней уравнения cos x = а при

    а = 0, а = -1, а = 1, которые можно легко получить, используя как ори-ентир единичную окружность.

    Поскольку косинус равен абсциссе соответствующей точки единичной окружности, получаем, что cos x = 0 тогда и только тогда, когда соответ-ствующей точкой единичной окружности является точка A или точка B.

    Аналогично cos x = 1 тогда и только тогда, когда соответствующей точкой единичной окружности является точка C, следовательно,

    x = 2πп, k € Z.

    Также cos х = —1 тогда и только тогда, когда соответствующей точкой единичной окружности является точка D, таким образом, х = п + 2пn,

    Уравнение sin (x) = a

    Объяснение и обоснование

    1. Корни уравнения sinx = а. При | а | > 1 уравнение не имеет корней, по-скольку | sinx | 1 или при а

    Видеокурс «Получи пятерку» включает все темы, необходимые для успешной сдачи ЕГЭ по математике на 60-65 баллов. Полностью все задачи 1-13 Профильного ЕГЭ по математике. Подходит также для сдачи Базового ЕГЭ по математике. Если вы хотите сдать ЕГЭ на 90-100 баллов, вам надо решать часть 1 за 30 минут и без ошибок!

    Курс подготовки к ЕГЭ для 10-11 класса, а также для преподавателей. Все необходимое, чтобы решить часть 1 ЕГЭ по математике (первые 12 задач) и задачу 13 (тригонометрия). А это более 70 баллов на ЕГЭ, и без них не обойтись ни стобалльнику, ни гуманитарию.

    Вся необходимая теория. Быстрые способы решения, ловушки и секреты ЕГЭ. Разобраны все актуальные задания части 1 из Банка заданий ФИПИ. Курс полностью соответствует требованиям ЕГЭ-2018.

    Курс содержит 5 больших тем, по 2,5 часа каждая. Каждая тема дается с нуля, просто и понятно.

    Сотни заданий ЕГЭ. Текстовые задачи и теория вероятностей. Простые и легко запоминаемые алгоритмы решения задач. Геометрия. Теория, справочный материал, разбор всех типов заданий ЕГЭ. Стереометрия. Хитрые приемы решения, полезные шпаргалки, развитие пространственного воображения. Тригонометрия с нуля — до задачи 13. Понимание вместо зубрежки. Наглядное объяснение сложных понятий. Алгебра. Корни, степени и логарифмы, функция и производная. База для решения сложных задач 2 части ЕГЭ.

    Соблюдение Вашей конфиденциальности важно для нас. По этой причине, мы разработали Политику Конфиденциальности, которая описывает, как мы используем и храним Вашу информацию. Пожалуйста, ознакомьтесь с нашими правилами соблюдения конфиденциальности и сообщите нам, если у вас возникнут какие-либо вопросы.

    Сбор и использование персональной информации

    Под персональной информацией понимаются данные, которые могут быть использованы для идентификации определенного лица либо связи с ним.

    От вас может быть запрошено предоставление вашей персональной информации в любой момент, когда вы связываетесь с нами.

    Ниже приведены некоторые примеры типов персональной информации, которую мы можем собирать, и как мы можем использовать такую информацию.

    Какую персональную информацию мы собираем:

    • Когда вы оставляете заявку на сайте, мы можем собирать различную информацию, включая ваши имя, номер телефона, адрес электронной почты и т.д.

    Как мы используем вашу персональную информацию:

    • Собираемая нами персональная информация позволяет нам связываться с вами и сообщать об уникальных предложениях, акциях и других мероприятиях и ближайших событиях.
    • Время от времени, мы можем использовать вашу персональную информацию для отправки важных уведомлений и сообщений.
    • Мы также можем использовать персональную информацию для внутренних целей, таких как проведения аудита, анализа данных и различных исследований в целях улучшения услуг предоставляемых нами и предоставления Вам рекомендаций относительно наших услуг.
    • Если вы принимаете участие в розыгрыше призов, конкурсе или сходном стимулирующем мероприятии, мы можем использовать предоставляемую вами информацию для управления такими программами.

    Раскрытие информации третьим лицам

    Мы не раскрываем полученную от Вас информацию третьим лицам.

    Исключения:

    • В случае если необходимо — в соответствии с законом, судебным порядком, в судебном разбирательстве, и/или на основании публичных запросов или запросов от государственных органов на территории РФ — раскрыть вашу персональную информацию. Мы также можем раскрывать информацию о вас если мы определим, что такое раскрытие необходимо или уместно в целях безопасности, поддержания правопорядка, или иных общественно важных случаях.
    • В случае реорганизации, слияния или продажи мы можем передать собираемую нами персональную информацию соответствующему третьему лицу – правопреемнику.

    Защита персональной информации

    Мы предпринимаем меры предосторожности — включая административные, технические и физические — для защиты вашей персональной информации от утраты, кражи, и недобросовестного использования, а также от несанкционированного доступа, раскрытия, изменения и уничтожения.

    Соблюдение вашей конфиденциальности на уровне компании

    Для того чтобы убедиться, что ваша персональная информация находится в безопасности, мы доводим нормы соблюдения конфиденциальности и безопасности до наших сотрудников, и строго следим за исполнением мер соблюдения конфиденциальности.

    Основными методами решения тригонометрических уравнений являются: сведение уравнений к простейшим (с использованием тригонометрических формул), введение новых переменных, разложение на множители. Рассмотрим их применение на примерах. Обратите внимание на оформление записи решений тригонометрических уравнений.

    Необходимым условием успешного решения тригонометрических уравнений является знание тригонометрических формул (тема 13 работы 6).

    Примеры.

    1. Уравнения, сводящиеся к простейшим.

    1) Решить уравнение

    Решение:

    Ответ:

    2) Найти корни уравнения

    (sinx + cosx) 2 = 1 – sinxcosx, принадлежащие отрезку .

    Решение:

    Ответ:

    2. Уравнения, сводящиеся к квадратным.

    1) Решить уравнение 2 sin 2 x – cosx –1 = 0.

    Решение: Используя формулу sin 2 x = 1 – cos 2 x, получаем

    Ответ:

    2) Решить уравнение cos 2x = 1 + 4 cosx.

    Решение: Используя формулу cos 2x = 2 cos 2 x – 1, получаем

    Ответ:

    3) Решить уравнение tgx – 2ctgx + 1 = 0

    Решение:

    Ответ:

    3. Однородные уравнения

    1) Решить уравнение 2sinx – 3cosx = 0

    Решение: Пусть cosx = 0, тогда 2sinx = 0 и sinx = 0 – противоречие с тем, что sin 2 x + cos 2 x = 1. Значит cosx ≠ 0 и можно поделить уравнение на cosx. Получим

    Ответ:

    2) Решить уравнение 1 + 7 cos 2 x = 3 sin 2x

    Решение:

    Используем формулы 1 = sin 2 x + cos 2 x и sin 2x = 2 sinxcosx, получим

    sin 2 x + cos 2 x + 7cos 2 x = 6sinxcosx
    sin 2 x – 6sinxcosx+ 8cos 2 x = 0

    Пусть cosx = 0, тогда sin 2 x = 0 и sinx = 0 – противоречие с тем, что sin 2 x + cos 2 x = 1.
    Значит cosx ≠ 0 и можно поделить уравнение на cos 2 x. Получим

    tg 2 x – 6 tgx + 8 = 0
    Обозначим tgx = y
    y 2 – 6 y + 8 = 0
    y 1 = 4; y 2 = 2
    а) tgx = 4, x= arctg4 + 2 k , k
    б) tgx = 2, x= arctg2 + 2 k , k .

    Ответ: arctg4 + 2 k , arctg2 + 2 k, k

    4. Уравнения вида a sinx + b cosx = с, с ≠ 0.

    1) Решить уравнение .

    Решение:

    Ответ:

    5. Уравнения, решаемые разложением на множители.

    1) Решить уравнение sin2x – sinx = 0.

    Корнем уравнения f ( х ) = φ ( х ) может служить только число 0. Проверим это:

    cos 0 = 0 + 1 – равенство верно.

    Число 0 единственный корень данного уравнения.

    Ответ: 0.

    Формулы по тригонометрии уравнения. Простейшие тригонометрические уравнения

    Соблюдение Вашей конфиденциальности важно для нас. По этой причине, мы разработали Политику Конфиденциальности, которая описывает, как мы используем и храним Вашу информацию. Пожалуйста, ознакомьтесь с нашими правилами соблюдения конфиденциальности и сообщите нам, если у вас возникнут какие-либо вопросы.

    Сбор и использование персональной информации

    Под персональной информацией понимаются данные, которые могут быть использованы для идентификации определенного лица либо связи с ним.

    От вас может быть запрошено предоставление вашей персональной информации в любой момент, когда вы связываетесь с нами.

    Ниже приведены некоторые примеры типов персональной информации, которую мы можем собирать, и как мы можем использовать такую информацию.

    Какую персональную информацию мы собираем:

    • Когда вы оставляете заявку на сайте, мы можем собирать различную информацию, включая ваши имя, номер телефона, адрес электронной почты и т.д.

    Как мы используем вашу персональную информацию:

    • Собираемая нами персональная информация позволяет нам связываться с вами и сообщать об уникальных предложениях, акциях и других мероприятиях и ближайших событиях.
    • Время от времени, мы можем использовать вашу персональную информацию для отправки важных уведомлений и сообщений.
    • Мы также можем использовать персональную информацию для внутренних целей, таких как проведения аудита, анализа данных и различных исследований в целях улучшения услуг предоставляемых нами и предоставления Вам рекомендаций относительно наших услуг.
    • Если вы принимаете участие в розыгрыше призов, конкурсе или сходном стимулирующем мероприятии, мы можем использовать предоставляемую вами информацию для управления такими программами.

    Раскрытие информации третьим лицам

    Мы не раскрываем полученную от Вас информацию третьим лицам.

    Исключения:

    • В случае если необходимо — в соответствии с законом, судебным порядком, в судебном разбирательстве, и/или на основании публичных запросов или запросов от государственных органов на территории РФ — раскрыть вашу персональную информацию. Мы также можем раскрывать информацию о вас если мы определим, что такое раскрытие необходимо или уместно в целях безопасности, поддержания правопорядка, или иных общественно важных случаях.
    • В случае реорганизации, слияния или продажи мы можем передать собираемую нами персональную информацию соответствующему третьему лицу – правопреемнику.

    Защита персональной информации

    Мы предпринимаем меры предосторожности — включая административные, технические и физические — для защиты вашей персональной информации от утраты, кражи, и недобросовестного использования, а также от несанкционированного доступа, раскрытия, изменения и уничтожения.

    Соблюдение вашей конфиденциальности на уровне компании

    Для того чтобы убедиться, что ваша персональная информация находится в безопасности, мы доводим нормы соблюдения конфиденциальности и безопасности до наших сотрудников, и строго следим за исполнением мер соблюдения конфиденциальности.

    Урок и презентация на тему: «Решение простейших тригонометрических уравнений»

    Дополнительные материалы
    Уважаемые пользователи, не забывайте оставлять свои комментарии, отзывы, пожелания! Все материалы проверены антивирусной программой.

    Пособия и тренажеры в интернет-магазине «Интеграл» для 10 класса от 1С
    Решаем задачи по геометрии. Интерактивные задания на построение в пространстве
    Программная среда «1С: Математический конструктор 6.1»

    Что будем изучать:
    1. Что такое тригонометрические уравнения?

    3. Два основных метода решения тригонометрических уравнений.
    4. Однородные тригонометрические уравнения.
    5. Примеры.

    Что такое тригонометрические уравнения?

    Ребята, мы с вами изучили уже арксинуса, арккосинус, арктангенс и арккотангенс. Теперь давайте посмотрим на тригонометрические уравнения в общем.

    Тригонометрические уравнения – уравнения в котором переменная содержится под знаком тригонометрической функции.

    Повторим вид решения простейших тригонометрических уравнений:

    1)Если |а|≤ 1, то уравнение cos(x) = a имеет решение:

    X= ± arccos(a) + 2πk

    2) Если |а|≤ 1, то уравнение sin(x) = a имеет решение:

    3) Если |а| > 1, то уравнение sin(x) = a и cos(x) = a не имеют решений 4) Уравнение tg(x)=a имеет решение: x=arctg(a)+ πk

    5) Уравнение ctg(x)=a имеет решение: x=arcctg(a)+ πk

    Для всех формул k- целое число

    Простейшие тригонометрические уравнения имеют вид: Т(kx+m)=a, T- какая либо тригонометрическая функция.

    Пример.

    Решить уравнения: а) sin(3x)= √3/2

    Решение:

    А) Обозначим 3x=t, тогда наше уравнение перепишем в виде:

    Решение этого уравнения будет: t=((-1)^n)arcsin(√3 /2)+ πn.n – минус один в степени n.

    Ещё примеры тригонометрических уравнений.

    Решить уравнения: а) cos(x/5)=1 б)tg(3x- π/3)= √3

    Решение:

    А) В этот раз перейдем непосредственно к вычислению корней уравнения сразу:

    X/5= ± arccos(1) + 2πk. Тогда x/5= πk => x=5πk

    Ответ: x=5πk, где k – целое число.

    Б) Запишем в виде: 3x- π/3=arctg(√3)+ πk. Мы знаем что: arctg(√3)= π/3

    3x- π/3= π/3+ πk => 3x=2π/3 + πk => x=2π/9 + πk/3

    Ответ: x=2π/9 + πk/3, где k – целое число.

    Решить уравнения: cos(4x)= √2/2. И найти все корни на отрезке .

    Решение:

    Решим в общем виде наше уравнение: 4x= ± arccos(√2/2) + 2πk

    4x= ± π/4 + 2πk;

    X= ± π/16+ πk/2;

    Теперь давайте посмотрим какие корни попадут на наш отрезок. При k При k=0, x= π/16, мы попали в заданный отрезок .
    При к=1, x= π/16+ π/2=9π/16, опять попали.
    При k=2, x= π/16+ π=17π/16, а тут вот уже не попали, а значит при больших k тоже заведомо не будем попадать.

    Ответ: x= π/16, x= 9π/16

    Два основных метода решения.

    Мы рассмотрели простейшие тригонометрические уравнения, но существуют и более сложные. Для их решения применяют метод ввода новой переменной и метод разложения на множители. Давайте рассмотрим примеры.

    Решим уравнение:

    Решение:
    Для решения нашего уравнения воспользуемся методом ввода новой переменной, обозначим: t=tg(x).

    В результате замены получим: t 2 + 2t -1 = 0

    Найдем корни квадратного уравнения: t=-1 и t=1/3

    Тогда tg(x)=-1 и tg(x)=1/3, получили простейшее тригонометрическое уравнение, найдем его корни.

    X=arctg(-1) +πk= -π/4+πk; x=arctg(1/3) + πk.

    Ответ: x= -π/4+πk; x=arctg(1/3) + πk.

    Пример решения уравнения

    Решить уравнений: 2sin 2 (x) + 3 cos(x) = 0

    Решение:

    Воспользуемся тождеством: sin 2 (x) + cos 2 (x)=1

    Наше уравнение примет вид:2-2cos 2 (x) + 3 cos (x) = 0

    2 cos 2 (x) — 3 cos(x) -2 = 0

    Введем замену t=cos(x): 2t 2 -3t — 2 = 0

    Решением нашего квадратного уравнения являются корни: t=2 и t=-1/2

    Тогда cos(x)=2 и cos(x)=-1/2.

    Т.к. косинус не может принимать значения больше единицы, то cos(x)=2 не имеет корней.

    Для cos(x)=-1/2: x= ± arccos(-1/2) + 2πk; x= ±2π/3 + 2πk

    Ответ: x= ±2π/3 + 2πk

    Однородные тригонометрические уравнения.

    Определение: Уравнение вида a sin(x)+b cos(x) называются однородными тригонометрическими уравнениями первой степени.

    Уравнения вида

    однородными тригонометрическими уравнениями второй степени.

    Для решения однородного тригонометрического уравнения первой степени разделим его на cos(x): Делить на косинус нельзя если он равен нулю, давайте убедимся что это не так:
    Пусть cos(x)=0, тогда asin(x)+0=0 => sin(x)=0, но синус и косинус одновременно не равны нулю, получили противоречие, поэтому можно смело делить на ноль.

    Решить уравнение:
    Пример: cos 2 (x) + sin(x) cos(x) = 0

    Решение:

    Вынесем общий множитель: cos(x)(c0s(x) + sin (x)) = 0

    Тогда нам надо решить два уравнения:

    Cos(x)=0 и cos(x)+sin(x)=0

    Cos(x)=0 при x= π/2 + πk;

    Рассмотрим уравнение cos(x)+sin(x)=0 Разделим наше уравнение на cos(x):

    1+tg(x)=0 => tg(x)=-1 => x=arctg(-1) +πk= -π/4+πk

    Ответ: x= π/2 + πk и x= -π/4+πk

    Как решать однородные тригонометрические уравнения второй степени?
    Ребята, придерживайтесь этих правил всегда!

    1. Посмотреть чему равен коэффициент а, если а=0 то тогда наше уравнение примет вид cos(x)(bsin(x)+ccos(x)), пример решения которого на предыдущем слайде

    2. Если a≠0, то нужно поделить обе части уравнения на косинус в квадрате, получим:


    Делаем замену переменной t=tg(x) получаем уравнение:

    Решить пример №:3

    Решить уравнение:
    Решение:

    Разделим обе части уравнения на косинус квадрат:

    Делаем замену переменной t=tg(x): t 2 + 2 t — 3 = 0

    Найдем корни квадратного уравнения: t=-3 и t=1

    Тогда: tg(x)=-3 => x=arctg(-3) + πk=-arctg(3) + πk

    Tg(x)=1 => x= π/4+ πk

    Ответ: x=-arctg(3) + πk и x= π/4+ πk

    Решить пример №:4

    Решить уравнение:

    Решение:
    Преобразуем наше выражение:


    Решать такие уравнение мы умеем: x= — π/4 + 2πk и x=5π/4 + 2πk

    Ответ: x= — π/4 + 2πk и x=5π/4 + 2πk

    Решить пример №:5

    Решить уравнение:

    Решение:
    Преобразуем наше выражение:


    Введем замену tg(2x)=t:2 2 — 5t + 2 = 0

    Решением нашего квадратного уравнения будут корни: t=-2 и t=1/2

    Тогда получаем: tg(2x)=-2 и tg(2x)=1/2
    2x=-arctg(2)+ πk => x=-arctg(2)/2 + πk/2

    2x= arctg(1/2) + πk => x=arctg(1/2)/2+ πk/2

    Ответ: x=-arctg(2)/2 + πk/2 и x=arctg(1/2)/2+ πk/2

    Задачи для самостоятельного решения.

    1) Решить уравнение

    А) sin(7x)= 1/2 б) cos(3x)= √3/2 в) cos(-x) = -1 г) tg(4x) = √3 д) ctg(0.5x) = -1.7

    2) Решить уравнения: sin(3x)= √3/2. И найти все корни на отрезке [π/2; π ].

    3) Решить уравнение: ctg 2 (x) + 2ctg(x) + 1 =0

    4) Решить уравнение: 3 sin 2 (x) + √3sin (x) cos(x) = 0

    5) Решить уравнение:3sin 2 (3x) + 10 sin(3x)cos(3x) + 3 cos 2 (3x) =0

    6)Решить уравнение:cos 2 (2x) -1 — cos(x) =√3/2 -sin 2 (2x)

    Простейшие тригонометрические уравнения решаются, как правило, по формулам. Напомню, что простейшими называются вот такие тригонометрические уравнения:

    sinx = а

    cosx = а

    tgx = а

    ctgx = а

    х — угол, который нужно найти,
    а — любое число.

    А вот и формулы, с помощью которых можно сразу записать решения этих простейших уравнений.

    Для синуса:

    Для косинуса:

    х = ± arccos a + 2π n, n ∈ Z

    Для тангенса:

    х = arctg a + π n, n ∈ Z

    Для котангенса:

    х = arcctg a + π n, n ∈ Z

    Собственно, это и есть теоретическая часть решения простейших тригонометрических уравнений. Причём, вся!) Совсем ничего. Однако, количество ошибок по этой теме просто зашкаливает. Особенно, при незначительном отклонении примера от шаблона. Почему?

    Да потому, что масса народу записывает эти буковки, не понимая их смысла совершенно! С опаской записывает, как бы чего не вышло…) С этим надо разобраться. Тригонометрия для людей, или люди для тригонометрии, в конце концов!?)

    Разберёмся?

    Один угол у нас будет равен arccos a, второй: -arccos a.

    И так будет получаться всегда. При любом а.

    Если не верите, наведите курсор мышки на картинку, или коснитесь рисунка на планшете.) Я изменил число а на какое-то отрицательное. Всё равно, один угол у нас получился arccos a, второй: -arccos a.

    Следовательно, ответ можно всегда записать в виде двух серий корней:

    х 1 = arccos a + 2π n, n ∈ Z

    х 2 = — arccos a + 2π n, n ∈ Z

    Объединяем эти две серии в одну:

    х= ± arccos а + 2π n, n ∈ Z

    И все дела. Получили общую формулу для решения простейшего тригонометрического уравнения с косинусом.

    Если вы понимаете, что это не какая-то сверхнаучная мудрость, а просто сокращённая запись двух серий ответов, вам и задания «С» будут по плечу. С неравенствами, с отбором корней из заданного интервала… Там ответ с плюсом/минусом не катит. А если отнестись к ответу делово, да разбить его на два отдельных ответа, всё и решается.) Собственно, для этого и разбираемся. Что, как и откуда.

    В простейшем тригонометрическом уравнении

    sinx = а

    тоже получается две серии корней. Всегда. И эти две серии тоже можно записать одной строчкой. Только эта строчка похитрее будет:

    х = (-1) n arcsin a + π n, n ∈ Z

    Но суть остаётся прежней. Математики просто сконструировали формулу, чтобы вместо двух записей серий корней, сделать одну. И всё!

    Проверим математиков? А то мало ли…)

    В предыдущем уроке подробно разобрано решение (безо всяких формул) тригонометрического уравнения с синусом:

    В ответе получились две серии корней:

    х 1 = π /6 + 2π n, n ∈ Z

    х 2 = 5π /6 + 2π n, n ∈ Z

    Если мы будем решать это же уравнение по формуле, получим ответ:

    х = (-1) n arcsin 0,5 + π n, n ∈ Z

    Вообще-то, это недоделанный ответ.) Ученик обязан знать, что arcsin 0,5 = π /6. Полноценный ответ будет:

    х = (-1) n π /6 + π n, n ∈ Z

    Тут возникает интересный вопрос. Ответ через х 1 ; х 2 (это правильный ответ!) и через одинокий х (и это правильный ответ!) — одно и то же, или нет? Сейчас узнаем.)

    Подставляем в ответ с х 1 значения n =0; 1; 2; и т.д., считаем, получаем серию корней:

    х 1 = π/6; 13π/6; 25π/6 и так далее.

    При такой же подстановке в ответ с х 2 , получаем:

    х 2 = 5π/6; 17π/6; 29π/6 и так далее.

    А теперь подставляем значения n (0; 1; 2; 3; 4…) в общую формулу для одинокого х . Т.е возводим минус один в нулевую степень, затем в первую, вторую, и т.д. Ну и, разумеется, во второе слагаемое подставляем 0; 1; 2 3; 4 и т.д. И считаем. Получаем серию:

    х = π/6; 5π/6; 13π/6; 17π/6; 25π/6 и так далее.

    Вот всё и видно.) Общая формула выдаёт нам точно такие же результаты, что и два ответа по отдельности. Только все сразу, по порядочку. Не обманули математики.)

    Формулы для решения тригонометрических уравнений с тангенсом и котангенсом тоже можно проверить. Но не будем.) Они и так простенькие.

    Я расписал всю эту подстановку и проверку специально. Здесь важно понять одну простую вещь: формулы для решения элементарных тригонометрических уравнений есть, всего лишь, краткая запись ответов. Для этой краткости пришлось вставить плюс/минус в решение для косинуса и (-1) n в решение для синуса.

    Эти вставки никак не мешают в заданиях, где нужно просто записать ответ элементарного уравнения. Но если надо решать неравенство, или далее нужно что-то делать с ответом: отбирать корни на интервале, проверять на ОДЗ и т.п, эти вставочки могут запросто выбить человека из колеи.

    И что делать? Да либо расписать ответ через две серии, либо решать уравнение/неравенство по тригонометрическому кругу. Тогда исчезают эти вставочки и жизнь становится легче.)

    Можно подвести итоги.

    Для решения простейших тригонометрических уравнений существуют готовые формулы ответов. Четыре штуки. Они хороши для мгновенной записи решения уравнения. Например, надо решить уравнения:

    sinx = 0,3

    Легко: х = (-1) n arcsin 0,3 + π n, n ∈ Z

    cosx = 0,2

    Без проблем: х = ± arccos 0,2 + 2π n, n ∈ Z

    tgx = 1,2

    Запросто: х = arctg 1,2 + π n, n ∈ Z

    ctgx = 3,7

    Одной левой: x= arcctg3,7 + π n, n ∈ Z

    cos x = 1,8

    Если вы, блистая знаниями, мгновенно пишете ответ:

    х= ± arccos 1,8 + 2π n, n ∈ Z

    то блистаете вы уже, это… того… из лужи.) Правильный ответ: решений нет. Не понимаете, почему? Прочитайте, что такое арккосинус. Кроме того, если в правой части исходного уравнения стоят табличные значения синуса, косинуса, тангенса, котангенса, — 1; 0; √3; 1/2; √3/2 и т.п. — ответ через арки будет недоделанным. Арки нужно обязательно перевести в радианы.

    А если уж вам попалось неравенство, типа

    то ответ в виде:

    х πn, n ∈ Z

    есть редкая ахинея, да…) Тут надо по тригонометрическому кругу решать. Чем мы и займёмся в соответствующей теме.

    Для тех, кто героически дочитал до этих строк. Я просто не могу не оценить ваши титанические усилия. Вам бонус.)

    Бонус:

    При записи формул в тревожной боевой обстановке, даже закалённые учёбой ботаны частенько путаются, где πn, а где 2π n. Вот вам простой приёмчик. Во всех формулах стоит πn. Кроме единственной формулы с арккосинусом. Там стоит 2πn. Два пиэн. Ключевое слово — два. В этой же единственной формуле стоят два знака в начале. Плюс и минус. И там, и там — два.

    Так что, если вы написали два знака перед арккосинусом, легче вспомнить, что в конце будет два пиэн. А ещё наоборот бывает. Пропустит человек знак ± , доберётся до конца, напишет правильно два пиэн, да и спохватится. Впереди-то два знака! Вернётся человек к началу, да ошибку-то и исправит! Вот так.)

    Если Вам нравится этот сайт…

    Кстати, у меня есть ещё парочка интересных сайтов для Вас.)

    Можно потренироваться в решении примеров и узнать свой уровень. Тестирование с мгновенной проверкой. Учимся — с интересом!)

    можно познакомиться с функциями и производными.

    Когда-то я стал свидетелем разговора двух абитуриентов:

    – Когда надо прибавить 2πn, а когда – πn? Никак не могу запомнить!

    – И у меня такая же проблема.

    Так и хотелось им сказать: «Не запоминать надо, а понимать!»

    Данная статья адресована прежде всего старшеклассникам и, надеюсь, поможет им с «пониманием» решать простейшие тригонометрические уравнения:

    Числовая окружность

    Наряду с понятием числовой прямой есть еще и понятие числовой окружности. Как мы знаем, в прямоугольной системе координат окружность,с центром в точке (0;0) и радиусом 1, называется единичной. Вообразим числовую прямую тонкой нитью и намотаем ее на эту окружность: начало отсчета (точку 0), приставим к «правой» точке единичной окружности, положительную полуось обмотаем против движения часовой стрелки, а отрицательную – по направлению (рис. 1). Такую единичную окружность называют числовой.

    Свойства числовой окружности

    • Каждое действительное число находится на одной точке числовой окружности.
    • На каждой точке числовой окружности находятся бесконечно много действительных чисел. Так как длина единичной окружности равна 2π, то разность между любыми двумя числами на одной точке окружности равна одному из чисел ±2π ; ±4π ; ±6π ; …

    Сделаем вывод: зная одно из чисел точки A, мы можем найти все числа точки A .

    Проведем диаметр АС (рис. 2). Так как x_0 – одно из чисел точки А, то числа x_0±π ; x_0±3π; x_0±5π; … и только они будут числами точки C. Выберем одно из этих чисел, скажем, x_0+π, и запишем с его помощью все числа точки C: x_C=x_0+π+2πk ,k∈Z. Отметим, что числа на точках A и C можно объединить в одну формулу: x_(A ; C)=x_0+πk ,k∈Z (при k = 0; ±2; ±4; … получим числа точки A, а при k = ±1; ±3; ±5; … – числа точки C).

    Сделаем вывод: зная одно из чисел на одной из точек A или C диаметра АС, мы можем найти все числа на этих точках.

    • Два противоположных числа находятся на симметричных относительно оси абсцисс точках окружности.

    Проведем вертикальную хорду АВ (рис. 2). Так как точки A и B симметричны относительно оси Ox, то число -x_0 находится на точке B и, значит, все числа точки B задаются формулой: x_B=-x_0+2πk ,k∈Z. Числа на точках A и B запишем одной формулой: x_(A ; B)=±x_0+2πk ,k∈Z. Сделаем вывод: зная одно из чисел на одной из точек A или B вертикальной хорды АВ, мы можем найти все числа на этих точках. Рассмотрим горизонтальную хорду AD и найдем числа точки D (рис. 2). Так как BD – диаметр и число -x_0 принадлежит точке В, то -x_0 + π одно из чисел точки D и, значит, все числа этой точки задаются формулой x_D=-x_0+π+2πk ,k∈Z.k∙x_0+πk ,k∈Z . (при k= 0; ±2; ±4; … получим числа точки A, а при k = ±1; ±3; ±5; … – числа точки D).

    Сделаем вывод: зная одно из чисел на одной из точек A или D горизонтальной хорды AD, мы можем найти все числа на этих точках.

    Шестнадцать основных точек числовой окружности

    На практике решение большинства простейших тригонометрических уравнений связано с шестнадцатью точками окружности (рис. 3). Что это за точки? Красные, синие и зеленые точки делят окружность на 12 равных частей. Так как длина полуокружности равна π, то длина дуги A1A2 равна π/2, длина дуги A1B1 равна π/6, а длина дуги A1C1 равна π/3.

    Теперь можем указать по одному числу на точках:

    π/3 на С1 и

    Вершины оранжевого квадрата – середины дуг каждой четверти, следовательно, длина дуги A1D1 равна π/4 и, значит, π/4 – одно из чисел точки D1. Воспользовавшись свойствами числовой окружности, мы можем записать с помощью формул все числа на всех отмеченных точках нашей окружности. На рисунке отмечены также и координаты этих точек (опустим описание их получения).

    Усвоив выше сказанное, мы имеем теперь достаточную подготовку для решения частных случаев (для девяти значений числа a) простейших уравнений.

    Решить уравнения

    1) sinx=1⁄(2) .

    – Что от нас требуется?

    Найти все те числа x, синус которых равен 1/2 .

    Вспомним определение синуса: sinx – ордината точки числовой окружности, на которой находится число x . На окружности имеем две точки, ордината которых равна 1/2 . Это концы горизонтальной хорды B1B2 . Значит, требование «решить уравнение sinx=1⁄2 » равнозначно требованию «найти все числа на точке B1 и все числа на точке B2».

    2) sinx=-√3⁄2 .

    Нам надо найти все числа на точках C4 и C3.

    3) sinx=1 . На окружности имеем только одну точку с ординатой 1 – точка A2 и, значит, нам надо найти только все числа этой точки.

    Ответ: x=π/2+2πk , k∈Z .

    4) sinx=-1 .

    Только точка A_4 имеет ординату -1. Все числа этой точки и будут конями уравнения.

    Ответ: x=-π/2+2πk , k∈Z .

    5) sinx=0 .

    На окружности имеем две точки с ординатой 0 – точки A1 и A3 . Можно указать числа на каждой из точек по отдельности, но, учитывая, что эти точки диаметрально противоположные, лучше объединить их в одну формулу: x=πk ,k∈Z .

    Ответ: x=πk ,k∈Z .

    6) cosx=√2⁄2 .

    Вспомним определение косинуса: cosx — абсцисса точки числовой окружности на которой находится число x. На окружности имеем две точки с абсциссой √2⁄2 – концы горизонтальной хорды D1D4 . Нам нужно найти все числа на этих точках. Запишем их, объединив в одну формулу.

    Ответ: x=±π/4+2πk , k∈Z .

    7) cosx=-1⁄2 .

    Надо найти числа на точках C_2 и C_3 .

    Ответ: x=±2π/3+2πk , k∈Z .

    10) cosx=0 .

    Только точки A2 и A4 имеют абсциссу 0, значит, все числа на каждой из этих точках и будут решениями уравнения.
    .

    Решениями уравнения системы являются числа на точках B_3 и B_4 .Неравенству cosxОтвет: x=-5π/6+2πk , k∈Z .

    Заметим,что при любом допустимом значении x второй множитель положителен и, следовательно,уравнение равносильно системе

    Решениями уравнения системы являются чила точек D_2 и D_3 . Числа точки D_2 не удовлетворяют неравенству sinx≤0,5 ,а числа точки D_3-удовлетворяют.


    blog.сайт, при полном или частичном копировании материала ссылка на первоисточник обязательна.

    Концепция решения тригонометрических уравнений.

    • Для решения тригонометрического уравнения преобразуйте его в одно или несколько основных тригонометрических уравнений. Решение тригонометрического уравнения в конечном итоге сводится к решению четырех основных тригонометрических уравнений.
  • Решение основных тригонометрических уравнений.

    • Существуют 4 вида основных тригонометрических уравнений:
    • sin x = a; cos x = a
    • tg x = a; ctg x = a
    • Решение основных тригонометрических уравнений подразумевает рассмотрение различных положений «х» на единичной окружности, а также использование таблицы преобразования (или калькулятора).
    • Пример 1. sin x = 0,866. Используя таблицу преобразования (или калькулятор), вы получите ответ: х = π/3. Единичная окружность дает еще один ответ: 2π/3. Запомните: все тригонометрические функции являются периодическими, то есть их значения повторяются. Например, периодичность sin x и cos x равна 2πn, а периодичность tg x и ctg x равна πn. Поэтому ответ записывается следующим образом:
    • x1 = π/3 + 2πn; x2 = 2π/3 + 2πn.
    • Пример 2. соs х = -1/2. Используя таблицу преобразования (или калькулятор), вы получите ответ: х = 2π/3. Единичная окружность дает еще один ответ: -2π/3.
    • x1 = 2π/3 + 2π; х2 = -2π/3 + 2π.
    • Пример 3. tg (x — π/4) = 0.
    • Ответ: х = π/4 + πn.
    • Пример 4. ctg 2x = 1,732.
    • Ответ: х = π/12 + πn.
  • Преобразования, используемые при решении тригонометрических уравнений.

    • Для преобразования тригонометрических уравнений используются алгебраические преобразования (разложение на множители, приведение однородных членов и т.д.) и тригонометрические тождества.
    • Пример 5. Используя тригонометрические тождества, уравнение sin x + sin 2x + sin 3x = 0 преобразуется в уравнение 4cos x*sin (3x/2)*cos (x/2) = 0. Таким образом, нужно решить следующие основные тригонометрические уравнения: cos x = 0; sin (3x/2) = 0; cos (x/2) = 0.
    • Нахождение углов по известным значениям функций.

      • Перед изучением методов решения тригонометрических уравнений вам необходимо научиться находить углы по известным значениям функций. Это можно сделать при помощи таблицы преобразования или калькулятора.
      • Пример: соs х = 0,732. Калькулятор даст ответ х = 42,95 градусов. Единичная окружность даст дополнительные углы, косинус которых также равен 0,732.
    • Отложите решение на единичной окружности.

      • Вы можете отложить решения тригонометрического уравнения на единичной окружности. Решения тригонометрического уравнения на единичной окружности представляют собой вершины правильного многоугольника.
      • Пример: Решения x = π/3 + πn/2 на единичной окружности представляют собой вершины квадрата.
      • Пример: Решения x = π/4 + πn/3 на единичной окружности представляют собой вершины правильного шестиугольника.
    • Методы решения тригонометрических уравнений.

      • Если данное тригонометрическое уравнение содержит только одну тригонометрическую функцию, решите это уравнение как основное тригонометрическое уравнение. Если данное уравнение включает две или более тригонометрические функции, то существуют 2 метода решения такого уравнения (в зависимости от возможности его преобразования).
      • Преобразуйте данное уравнение в уравнение вида: f(x)*g(x)*h(x) = 0, где f(x), g(x), h(x) — основные тригонометрические уравнения.
      • Пример 6. 2cos x + sin 2x = 0. (0
      • Решение. Используя формулу двойного угла sin 2x = 2*sin х*соs х, замените sin 2x.
      • 2соs х + 2*sin х*соs х = 2cos х*(sin х + 1) = 0. Теперь решите два основных тригонометрических уравнения: соs х = 0 и (sin х + 1) = 0.2 — 1) = 0. Теперь найдите t, а затем найдите х для t = tg х.
  • методика преподавания – статья – Корпорация Российский учебник (издательство Дрофа – Вентана)

    1. Синус и косинус любого угла (пропедевтика к изучению тригонометрических уравнений)

    Пример задания. Найти приближенно углы, косинусы которых равны 0,8.

    Решение. Косинус — это абсцисса соответствующей точки единичной окружности. Все точки с абсциссами, равными 0,8, принадлежат прямой, параллельной оси ординат и проходящей через точку C(0,8; 0). Эта прямая пересекает единичную окружность в двух точках: Pα° и Pβ°, симметричных относительно оси абсцисс.

    С помощью транспортира находим, что угол α° приближенно равен 37°. Значит, общий вид углов поворота с конечной точкой Pα°:

    α° ≈ 37° + 360°n, где n — любое целое число.

    В силу симметрии относительно оси абсцисс точка Pβ° — конечная точка поворота на угол –37°. Значит, для нее общий вид углов поворота:

    β° ≈ –37° + 360°n, где — любое целое число.

    Ответ: 37° + 360°n, –37° + 360°n, где n— любое целое число.

    Пример задания. Найти углы, синусы которых равны 0,5.

    Решение. Синус — это ордината соответствующей точки единичной окружности. Все точки с ординатами, равными 0,5, принадлежат прямой, параллельной оси абсцисс и проходящей через точку D(0; 0,5).

    Эта прямая пересекает единичную окружность в двух точках: Pφ и Pπ–φ, симметричных относительно оси ординат. В прямоугольном треугольнике OKPφ катет KPφ равен половине гипотенузы OPφ, значит, 

    Общий вид углов поворота с конечной точкой Pφ:


    где n — любое целое число. Общий вид углов поворота с конечной точкой Pπ–φ:


    где n — любое целое число.

    Ответ:  где n — любое целое число.

    2. Тангенс и котангенс любого угла (пропедевтика к изучению тригонометрических уравнений)

    Пример 2. Найти общий вид углов, тангенс которых равен –1,2.

    Пример задания. Найти общий вид углов, тангенс которых равен –1,2.

    Решение. Отметим на оси тангенсов точку C с ординатой, равной –1,2, и проведем прямую OC. Прямая OC пересекает единичную окружность в точках Pα° и Pβ° — концах одного и того же диаметра. Углы, соответствующие этим точкам, отличаются друг от друга на целое число полуоборотов, т.е. на 180°n (n — целое число). С помощью транспортира находим, что угол Pα° OP0 равен –50°. Значит, общий вид углов, тангенс которых равен –1,2, следующий: –50° + 180°n (n — целое число)

    Ответ: –50° + 180°n, n ∈ Z.

    По синусу и косинусу углов 30°, 45° и 60° легко найти их тангенсы и котангенсы. Например,

    Перечисленные углы довольно часто встречаются в разных задачах, поэтому полезно запомнить значения тангенса и котангенса этих углов.

    α°

    30°

    45°

    60°

    φ рад

     

    tg φ

    1

    ctg φ

       

    1

       

    Математика: алгебра и начала математического анализа, геометрия. Алгебра и начала математического анализа. 11 класс. Базовый уровень

    Учебник входит в УМК по математике для 10–11 классов, изучающих предмет на базовом уровне. Теоретический материал разделен на обязательный и дополнительный, система заданий дифференцирована по уровню сложности, каждый пункт главы завершается контрольными вопросами и заданиями, а каждая глава — домашней контрольной работой. В учебник включены темы проектов и сделаны ссылки на интернет-ресурсы.

    Купить

    3. Простейшие тригонометрические уравнения

    Вводятся обозначения: arcsin α, arccos α, arctg α, arcctg α. Не рекомендуется торопиться с введением объединенной формулы. Две серии корней значительно удобнее записывать, особенно, когда нужно отбирать корни на интервале.

    tg φ = α,

    φ = arctg α + πn, n ∊ Z,

    т.е. arctg α — угол из промежутка  тангенс которого равен α,

    tg (arctg α) = α.

          

    ctg φ = α,

    φ = arcctg α + πn, n ∊ Z,

    0 < arcctg α < π,

    т.е. arcctg α — угол из промежутка (0; π), котангенс которого равен α,

    ctg (arcctg α) = α.

    При изучении темы «простейшие тригонометрические уравнения», уравнения чаще всего сводятся к квадратам.

    4. Формулы приведения

    Формулы приведения являются тождествами, т. е. они верны для любых допустимых значений φ. Анализируя полученную таблицу, можно заметить, что:

    1) знак в правой части формулы совпадает со знаком приводимой функции в соответствующей четверти, если считать φ острым углом;

    2) название меняют только функции углов  и 

     α

    φ + 2πn

    – φ

    π – φ

    π + φ

    sin α

    sin φ

    – sin φ

    sin φ

    – sin φ

    cos α

    cos φ

    cos φ

    – cos φ

    – cos φ

    tg α

    tg φ

    – tg φ

    – tg φ

    tg φ

    ctg α

    ctg φ

    – ctg φ

    – ctg φ

    ctg φ

    α

                   

    sin α

    cos φ

    cos φ

    – cos φ

    – cos φ

    cos α

    sin φ

    – sin φ

    – sin φ

    sin φ

    tg α

    ctg φ

    – ctg φ

    ctg φ

    – ctg φ

    ctg α

    tg φ

    — tg φ

    tg φ

    – tg φ

    5. Свойства и график функции

    y = sin x

    Простейшие тригонометрические неравенства решаются либо по графику, либо на окружности. При решении тригонометрического неравенства на окружности важно не перепутать, какую точку указывать первой.

    Что ещё почитать?

    6. Свойства и график функции

    y = cos x

    Задачу построения графика функции y = cosможно свести к построению графика функции y = sin x. Действительно, поскольку  график функции y = cos x можно получить из графика функции y = sin x сдвигом последнего вдоль оси абсцисс влево на

    7. Свойства и графики функций

    y = tg x и y = ctg x

    Область определения функции y = tg x включает в себя все числа, кроме чисел вида  где n Z. Как и при построении синусоиды, сначала постараемся получить график функции y = tg x на промежутке

    В левом конце этого промежутка тангенс равен нулю, а при приближении к правому концу значения тангенса неограниченно увеличиваются. Графически это выглядит так, как будто график функции y = tg x прижимается к прямой  уходя вместе с ней неограниченно вверх.

    8. Зависимости между тригонометрическими функциями одного и того же аргумента

    Равенства  и  выражают соотношения между тригонометрическими функциями одного и того же аргумента φ. С их помощью, зная синус и косинус некоторого угла, можно найти его тангенс и котангенс. Из этих равенств легко получить, что тангенс и котангенс связаны между собой следующим равенством.

    tg φ · ctg φ = 1

    Есть и другие зависимости между тригонометрическими функциями.

    Уравнение единичной окружности с центром в начале координат x2 + y2= 1 связывает абсциссу и ординату любой точки этой окружности.

    Основное тригонометрическое тождество

    cos2 φ + sin2 φ = 1

    9. Синус и косинус суммы и разности двух углов

    Формула косинуса суммы

    cos (α + β) = cos α cos β – sin α sin β

    Формула косинуса разности

    cos (α – β) = cos α cos β + sin α sin β

    Формула синуса разности

    sin (α – β) = sin α cos β – cos α sin β

    Формула синуса суммы

    sin (α + β) = sin α cos β + cos α sin β

    10. Тангенс суммы и тангенс разности двух углов

    Формула тангенса суммы

    Формула тангенса разности

    Математика: алгебра и начала математического анализа, геометрия. Алгебра и начала математического анализа. 11 класс. Углубленный уровень. Учебник

    Учебник входит в УМК по математике для 10–11 классов, изучающих предмет на базовом уровне. Теоретический материал разделен на обязательный и дополнительный, система заданий дифференцирована по уровню сложности, каждый пункт главы завершается контрольными вопросами и заданиями, а каждая глава — домашней контрольной работой. В учебник включены темы проектов и сделаны ссылки на интернет-ресурсы.

    Купить

    11. Тригонометрические функции двойного угла

    Формула тангенса двойного угла

    cos2α = 1 – 2sin2α cos2α = 2cos2α – 1

    Пример задания. Решить уравнение

    Решение.

    Понизим степень еще раз:

    Ответ:

    12. Преобразование произведения тригонометрических функций в сумму. Обратное преобразование

    Основные формулы
    Переход от суммы к произведению

    Переход от произведения к сумме

    13. Решение тригонометрических уравнений

    В большинстве случаев исходное уравнение в процессе решения сводится к простейшим тригонометрическим уравнениям. Однако для тригонометрических уравнений не существует единого метода решения. В каждом конкретном случае успех зависит от знания тригонометрических формул и от умения выбрать из них нужные. При этом обилие различных формул иногда делает этот выбор довольно трудным.

    Уравнения, сводящиеся к квадратам

    Пример задания. Решить уравнение 2 cos2x + 3 sinx = 0

    Решение. С помощью основного тригонометрического тождества это уравнение можно свести к квадратному относительно sinx:

    2cos2x + 3sinx = 0, 2(1 – sin2x) + 3sinx = 0,

    2 – 2sin2x + 3sinx = 0, 2sin2x – 3sinx – 2 = 0

    Введем новую переменную y = sin x, тогда уравнение примет вид: 2y2 – 3y – 2 = 0.

    Корни этого уравнения y1 = 2, y2 = –0,5.

    Возвращаемся к переменной x и получаем простейшие тригонометрические уравнения:

    1) sin x = 2 – это уравнение не имеет корней, так как sin x < 2 при любом значении x;

    2) sin x = –0,5, 

    Ответ:

    Однородные тригонометрические уравнения

    Пример задания. Решить уравнение 2sin2x – 3sinxcosx – 5cos2x = 0.

    Решение. Рассмотрим два случая:

    1) cosx = 0 и 2) cosx ≠ 0.

    Случай 1. Если cos x = 0, то уравнение принимает вид 2sin2x = 0, откуда sinx = 0. Но это равенство не удовлетворяет условию cosx = 0, так как ни при каком x косинус и синус одновременно в нуль не обращаются.

    Случай 2. Если cos x ≠ 0, то можно разделить уравнение на cos2x и получить 2tg2x – 3tg– 5 = 0. Вводя новую переменную y = tg x, получаем квадратное уравнение 2y2 – 3y — 5 = 0.

    Корни этого уравнения y1 = –1, y2 = 2,5.

    Возвращаемся к переменной x.

    tg x = 2,5,

    x = arctg 2,5 + πn, n ∈ Z.

    Ответ:

    Уравнение, левая часть которого — многочлен, каждый член которого имеет вторую степень, а правая — нуль, называют однородным уравнением второй степени относительно переменных и v.

    Обозначив в исходном уравнении sin x буквой u, а cos x буквой v, получим уравнение вида au2 + buv + cv2 = 0.

    Делением на v2 такое уравнение сводится к квадратному относительно

    Напоминаем, что апробировать учебник «Алгебра и начало математического анализа. 10 класс», как и многие другие издания, можно на платформе LECTA. Для этого воспользуйтесь предложением «5 учеников бесплатно».

    #ADVERTISING_INSERT#


    Тригонометрические уравнения — презентация онлайн

    1. Методика решения тригонометрических уравнений

    L/O/G/O

    2. Слово «тригонометрия» греческого происхождения. В пере-воде на русский язык оно означает «измерение треугольников». Как и все

    Слово «тригонометрия»
    греческого происхождения. В переводе на русский язык оно означает
    «измерение треугольников». Как и
    все разделы математики, зародившиеся в глубокой древности, тригонометрия возникла в результате
    попыток решить те задачи, с
    которыми человеку приходилось
    сталкиваться на практике.
    Основы тригонометрии, как и основы
    алгебры и начал анализа закладываются в
    школе. Тригонометрические функции
    начинают изучать в 8 классе на уроках
    геометрии и продолжают в 10-11 классах.
    Тригонометрические уравнения слишком
    разнообразны для того, чтобы попытаться
    дать их общую классификацию или общий
    метод решения. Мы можем указать лишь
    способы решения некоторых типов таких
    уравнений.
    Решение тригонометрических
    уравнений
    Для тригонометрических
    уравнений применимы общие методы
    решения (разложение на множители,
    замена переменной, функциональнографические) и равносильные
    преобразования общего характера.

    5. Методы решения тригонометрических уравнений

    Основные методы:
    • замена переменной,
    • разложение на множители,
    •однородные уравнения,
    прикладные методы:
    • по формулам преобразования суммы в произведение
    и произведения в сумму,
    • по формулам понижения степени,
    • универсальная тригонометрическая подстановка
    • введение вспомогательного угла,
    • умножение на некоторую тригонометрическую функцию.
    Проблемы ,возникающие при решении
    тригонометрических уравнений
    1.Потеря корней:
    делим на g(х).
    опасные формулы (универсальная подстановка).
    Этими операциями мы сужаем область определения.
    2. Лишние корни:
    возводим в четную степень.
    умножаем на g(х) (избавляемся от знаменателя).
    Этими операциями мы расширяем область
    определения.

    7. Наша задача: свести любое тригонометрическое уравнение к простейшему виду.

    Наша задача:
    свести любое
    тригон ометрическое
    урав н ен и е
    к п ростей шему вид у.
    Решение простейших
    тригонометрических
    уравнений

    9. Формулы корней простых тригонометрических уравнений

    1.cost = а , где |а| ≤ 1
    2.sint = а, где | а |≤ 1
    3. tgt = а, аЄR
    t = arctg а + πk‚ kЄZ
    или
    или
    4. ctgt = а, аЄR
    Частные случаи
    Частные случаи
    1)cost=0
    t = π/2+πk‚ kЄZ
    1)sint=0
    t = 0+πk‚ kЄZ
    2)cost=1
    t = 0+2πk‚ kЄZ
    2)sint=1
    t = π/2+2πk‚ kЄZ
    3)cost = -1
    t = π+2πk‚ kЄZ
    3)sint = — 1
    t = — π/2+2πk‚ kЄZ
    t = arcctg а + πk‚ kЄZ
    • При повторении формул решения уравнений
    следует обратить внимание на то, что формулы
    задают множества чисел, которые образованы по
    закону арифметической прогрессии с разностью
    2π или π.
    • С другой стороны использование общей
    формулы серий решений не всегда является
    удобной при отборе корней, в частности, на
    числовой окружности. В этом случае как раз
    удобнее не объединять серии решений
    тригонометрических уравнений, а представлять
    их совокупностью, выделяя разность 2π
    соответствующих прогрессий.

    13. tg x и ctg x

    14. Решение простейших уравнений

    2) cos(x+π/3) = ½
    1) tg2x = -1
    2x = arctg (-1) + πk, kЄZ
    2x = -π/4 + πk, kЄZ
    x = -π/8 + πk/2, kЄZ
    Ответ: -π/8 + πk/2, kЄZ.
    x+π/3 = ±arccos1/2 + 2πk, kЄZ
    x+π/3 = ±π/3 + 2πk, kЄZ
    x = -π/3 ± π/3 + 2πk, kЄZ
    Ответ: -π/3 ± π/3 + 2πk, kЄZ
    3) sin(π – x/3) = 0
    упростим по формулам
    приведения
    sin(x/3) = 0
    частный случай
    x/3 = πk, kЄZ
    x = 3πk, kЄZ.
    Ответ: 3πk, kЄZ.
    РЕШЕНИЕ ТРИГОНОМЕТРИЧЕСКИХ УРАВНЕНИЙ
    РАЗЛОЖЕНИЕМ НА МНОЖИТЕЛИ.
    Метод разложения на множители заключается в следующем: если
    То всякое решение уравнения
    Является решением совокупности уравнений
    Обратное утверждение, неверно: не всякое решение совокупности уравнений (2)
    является решением уравнения (1). Это объясняется тем, что решения отдельных
    уравнений (2) могут не входить в область определения функции
    .Поэтому,
    если при решении тригонометрического уравнения методом разложения
    на множители, функции,
    входящие в уравнение, определены не для всех значений аргумента, после
    нахождения решения должна быть сделана проверка,
    чтобы исключить лишние корни. Можно поступать другим способом: находить
    область допустимых значений исходного уравнения и выбирать только те корни,
    которые входят в найденную область допустимых значений
    РЕШЕНИЕ ТРИГОНОМЕТРИЧЕСКИХ УРАВНЕНИЙ
    РАЗЛОЖЕНИЕМ НА МНОЖИТЕЛИ.
    РЕШЕНИЕ ТРИГОНОМЕТРИЧЕСКИХ УРАВНЕНИЙ ,
    СВОДЯЩИХСЯ К КВАДРАТНЫМ.
    Уравнения сводимые к квадратным
    a∙sin²x + b∙sinx + c=0
    Пусть sinx = p, где |p| ≤1, тогда
    a∙p² + b∙p + c = 0
    Найти корни, вернуться к замене и решить простые уравнения.
    При решении уравнений указанного типа в основном применяются следующие
    тригонометрические тождества:
    РЕШЕНИЕ ТРИГОНОМЕТРИЧЕСКИХ УРАВНЕНИЙ ,
    СВОДЯЩИХСЯ К КВАДРАТНЫМ.
    Пример 1. Решить уравнение
    Решение.
    2 sin2x + sinx — 1 = 0.
    Введём новую переменную t = sinx. Тогда данное
    уравнение примет вид
    2t2 + t — 1 = 0.
    Решим его: D = 1 + 8 = 9,
    1 3 1
    t1
    ,
    4
    2
    1 3
    t2
    1 .
    4
    Cледовательно,
    sinx = 1/2
    или
    sinx = -1.
    1) sinx = 1/2,
    1
    х ( 1) arcsin k , k Z ,
    2
    k
    x ( 1)
    k
    6
    k, k Z.
    2) sinx = -1,
    х
    2
    2 n, n Z .
    Ответ : ( 1)
    k
    6
    k, k Z ,
    2
    2 n, n Z .

    20. Решение уравнений, однородных относительно синуса и косинуса

    в которых сумма показателей степеней у sinx и cosx (степень уравнения) во
    всех членах уравнения одинакова. Например,

    22. Однородные тригонометрические уравнения

    ОДНОРОДНЫЕ ТРИГОНОМЕТРИЧЕСКИЕ УРАВНЕНИЯ
    2.Однородные
    1)Первой степени:
    a∙sinx + b∙cosx = 0
    Т.к. sinx и cosx одновременно не равны нулю, то разделим обе
    части уравнения на cosx. Получим: простое уравнение
    a∙tgx + b = 0 или tgx = m
    2)Второй степени:
    a∙sin²x + b∙sinx∙cosx + c∙cos²x = 0
    Разделим обе части на cos²x. Получим квадратное уравнение:
    a∙tg²x + b∙tgx + c = 0.
    В частности, уравнения вида
    приводятся к однородным путем представления правой части в виде:

    23. Решение уравнений с помощью введения вспомогательного аргумента.

    :
    РЕШЕНИЕ УРАВНЕНИЙ С ПОМОЩЬЮ ВВЕДЕНИЯ
    ВСПОМОГАТЕЛЬНОГО АРГУМЕНТА.
    Рассмотрим уравнение
    Разделим левую и правую часть уравнения на
    Так как
    то существует угол φ такой, что
    при этом
    Тогда уравнение примет вид
    будут не всегда равносильны.
    Отметим, что к выбору угла φ в задачах с параметрами нужно относиться внимательно:
    выбор
    и выбор
    Решите уравнения:
    Уравнения, линейные относительно
    sin x и cos x
    а sin x + в cos x = с.
    Если а=в=0, а с не равно 0, то уравнение теряет смысл;
    Если а=в=с=0, то х – любое действительное число, то есть уравнение
    обращается в тождество.
    Рассмотрим случаи, когда а,в,с не равны 0.
    Примеры:
    3 sin 5x — 4 cos 5x = 2
    2 sin x cos x 2
    2 sin 3x + 5 cos 3x = 8.
    Последнее уравнение не имеет решений, так как левая часть его не
    превосходит 7. Уравнения, этого вида можно решить многими способами:
    с помощью универсальной подстановки, выразив sin x и cos x через tgх ;
    сведением уравнения к однородному; введением вспомогательного
    аргумента и другими.
    2
    2
    2
    Решение этих уравнений существует при a b c

    27. Решение уравнений с применением формул понижения степени.

    :
    Решение уравнений с применением
    формул понижения степени.
    При решении широкого круга тригонометрических уравнений ключевую роль играют
    формулы понижения степени
    Решение уравнений с применением
    формул тройного аргумента.
    (1)
    (2)
    При решении ряда уравнений наряду с другими существенную роль играют формулы

    28. Решение уравнений методом универсальной подстановки.

    Тригонометрическое уравнение вида
    где R – рациональная функция,
    с помощью тригонометрических формул двойного и тройного аргумента, а также
    формул сложения можно свести к рациональному уравнению относительно аргументов
    после чего уравнение может быть сведено к рациональному уравнению относительно
    с помощью формул универсальной тригонометрической подстановки
    Следует отметить, что применение формул может приводить к сужению ОДЗ исходного
    уравнения, поскольку
    не определен в точках
    поэтому в таких случаях нужно проверять, являются ли углы
    корнями исходного уравнения.

    29. тригонометрические уравнения, содержащие знак модуля или знак корня.

    ТРИГОНОМЕТРИЧЕСКИЕ УРАВНЕНИЯ, СОДЕРЖАЩИЕ
    ЗНАК МОДУЛЯ ИЛИ ЗНАК КОРНЯ.
    Специфика тригонометрических уравнений, содержащих знак модуля или
    знак корня, состоит в том, что они сводятся к смешанным системам, где
    кроме уравнений нужно решать тригонометрические неравенства и из
    решений уравнений выбирать лишь те, которые удовлетворяют
    неравенствам.
    Решите уравнения:

    30. Использование ограниченности функций при решении тригонометрических уравнений.

    ИСПОЛЬЗОВАНИЕ ОГРАНИЧЕННОСТИ ФУНКЦИЙ ПРИ
    РЕШЕНИИ ТРИГОНОМЕТРИЧЕСКИХ УРАВНЕНИЙ.
    При решении некоторых тригонометрических уравнений часто используется свойство
    ограниченности функций
    , то есть следующие неравенства:
    и

    31. Функциональные методы решения тригонометрических и комбинированных уравнений.

    ФУНКЦИОНАЛЬНЫЕ МЕТОДЫ РЕШЕНИЯ
    ТРИГОНОМЕТРИЧЕСКИХ И КОМБИНИРОВАННЫХ
    УРАВНЕНИЙ.
    Не всякое уравнение f(x)=g(x) в результате преобразований может быть сведено к
    уравнению того или иного стандартного вида, для которого существует определенный
    метод решения. В таких случаях оказывается полезным использовать такие свойства
    функций f(x) и g(x), как монотонность, ограниченность, четность, периодичность и др. Так,
    если одна из функций убывает, а вторая возрастает на промежутке X, то при наличии у
    уравнения f(x)=g(x) корня на этом промежутке, этот корень единственный, и тогда его,
    например, можно найти подбором. Если, далее, функция f(x) на промежутке X ограничена
    сверху, причем
    , а функция g(x) ограничена снизу, причем
    то уравнение f(x)=g(x) равносильно системе уравнении
    Иногда для решения уравнения f(x)=g(x) можно построить графики функции y=f(x),
    y=g(x) и определить абсциссы точек пересечения. Также рассматривается
    применение производной для исследования тригонометрических уравнений.

    32. Способы отбора корней тригонометрических уравнений на заданном промежутке

    • Арифметический способ
    Перебор значений целочисленного параметра n и
    вычисление корней
    • Алгебраический способ
    Перебор значений целочисленного параметра n и
    вычисление корней
    • Геометрический способ
    Изображение корней на тригонометрической
    окружности с последующим отбором с учетом
    имеющихся ограничений

    33. Арифметический способ Перебор значений целочисленного параметра n и вычисление корней

    cos x a, a 1
    x arccos a 2 k , k Z
    x arccos a 2 k , k Z
    Решить уравнение
    Записать корни уравнения
    Разделить виды решения для
    x
    косинуса; подсчитать значения x
    k
    -2
    -1 0
    2 …
    arccos a 2 k
    arccos a 2 k
    при целых n до тех пор, пока
    значения x не выйдут за пределы
    sin x a, a 1
    данного отрезка.
    x ( 1) k arcsin a k , k Z
    Записать ответ.
    1
    x
    k
    ( 1) k arcsin a k
    -2 -1 0 1 2 …

    34. Алгебраический способ Решение неравенства относительно неизвестного параметра n и вычисление корней

    cos x a, a 1, c; d
    Записать двойное неравенство
    для неизвестного (x),
    соответственное данному отрезку
    или условию; решить уравнение.
    2. Для синуса и косинуса разбить
    решения на два.
    3. Подставить в неравенство
    вместо неизвестного (x)
    найденные решения и решить его
    относительно n.
    4. Учитывая, что n принадлежит
    Z, найти соответствующие
    неравенству значения n.
    5. Подставить полученные
    значения n в формулу корней.
    1.
    x arccos a 2 k , k Z
    x arccos a 2 k , k Z
    с arccos a 2 k d
    с arccos a 2 k d
    Т.к. k Z ,
    то k1 …; x1 …
    k2 …; x2 …
    sin x a, a 1, c; d
    x arcsin a 2 k , k Z
    x arcsin a 2 k , k Z
    с arcsin a 2 k d
    с arcsin a 2 k d
    Т.к. k Z ,
    то k1 …; x1 …
    k2 …; x2 …

    35. Геометрический способ Изображение корней на тригонометрической окружности с последующим отбором с учетом имеющихся ограничений

    cos x a, a 1, c; d
    На окружности
    1. Решить уравнение.
    2. Обвести дугу,
    соответствующую данному
    отрезку на окружности.
    3. Разделить виды решений для
    синуса и косинуса.
    4. Нанести решения уравнения
    на окружность.
    5. Выбрать решения, попавшие
    на обведенную дугу.
    x arccos a 2 k , k Z
    x arccos a 2 k , k Z
    y
    arccos a
    d
    а
    0
    -arccos a
    c
    x

    36. Геометрический способ Изображение корней на графике с последующим отбором с учетом имеющихся ограничений

    На графике
    1. Решить уравнение.
    2. Построить график
    данной функции, прямую у
    = а, на оси х отметить
    данный отрезок.
    3. Найти точки
    пересечения графиков.
    4. Выбрать решения,
    принадлежащие данному
    отрезку.
    sin x a, a 1, c; d
    x arcsin a 2 k , k Z
    x arcsin a 2 k , k Z
    y
    y=a
    a
    с
    arcsin a
    П-arcsin a
    d
    y = sin x
    x
    Пример: Найти все корни уравнения
    10 cos 2 (
    2
    которые удовлетворяют условию x [
    Решение.
    10sin2 x = – cos 2x + 3;
    10sin2 x = 2sin2 x – 1 + 3,
    8sin2 x = 2;
    1
    sin 2 x ;
    4
    1
    sin x ;
    2
    k
    x
    (
    1
    )
    k , k Z ,
    6
    x ( 1) m ( ) m, m Z ;
    6
    6
    n, n Z ;
    7
    2 x) 3,
    2
    2 19
    ;
    ].
    3 12
    y
    2
    5
    6
    6
    0
    7
    6
    С помощью числовой окружности получим:
    x
    x) sin(
    x
    3
    2
    6
    Выберем корни, удовлетворяющие условию задачи.
    Из первой серии: 2 n 19 , n Z ;
    3
    6
    12
    8 2 12 n 19 , n Z ;
    10 12n 17, n Z .
    Следовательно n=0 или n=1, то есть x ,
    6
    x 7 .
    6
    2
    19
    n
    ,n Z;
    3
    6
    12
    8 2 12 n 19 , n Z ;
    Из второй серии:
    6 12n 21, n Z .
    Следовательно n=0 или n=1, то есть
    x 6 ,
    x 5 .
    6
    Ответ : {
    5 7
    ;
    6 6
    ;
    6
    }.

    39. Спасибо за внимание!

    L/O/G/O

    Тригонометрические уравнения — формулы, решения, примеры. Тригонометрические уравнения Тригонометрические уравнения решаемые разложением на множители

    Основными методами решения тригонометрических уравнений являются: сведение уравнений к простейшим (с использованием тригонометрических формул), введение новых переменных, разложение на множители. Рассмотрим их применение на примерах. Обратите внимание на оформление записи решений тригонометрических уравнений.

    Необходимым условием успешного решения тригонометрических уравнений является знание тригонометрических формул (тема 13 работы 6).

    Примеры.

    1. Уравнения, сводящиеся к простейшим.

    1) Решить уравнение

    Решение:

    Ответ:

    2) Найти корни уравнения

    (sinx + cosx) 2 = 1 – sinxcosx, принадлежащие отрезку .

    Решение:

    Ответ:

    2. Уравнения, сводящиеся к квадратным.

    1) Решить уравнение 2 sin 2 x – cosx –1 = 0.

    Решение: Используя формулу sin 2 x = 1 – cos 2 x, получаем

    Ответ:

    2) Решить уравнение cos 2x = 1 + 4 cosx.

    Решение: Используя формулу cos 2x = 2 cos 2 x – 1, получаем

    Ответ:

    3) Решить уравнение tgx – 2ctgx + 1 = 0

    Решение:

    Ответ:

    3. Однородные уравнения

    1) Решить уравнение 2sinx – 3cosx = 0

    Решение: Пусть cosx = 0, тогда 2sinx = 0 и sinx = 0 – противоречие с тем, что sin 2 x + cos 2 x = 1. Значит cosx ≠ 0 и можно поделить уравнение на cosx. Получим

    Ответ:

    2) Решить уравнение 1 + 7 cos 2 x = 3 sin 2x

    Решение:

    Используем формулы 1 = sin 2 x + cos 2 x и sin 2x = 2 sinxcosx, получим

    sin 2 x + cos 2 x + 7cos 2 x = 6sinxcosx
    sin 2 x – 6sinxcosx+ 8cos 2 x = 0

    Пусть cosx = 0, тогда sin 2 x = 0 и sinx = 0 – противоречие с тем, что sin 2 x + cos 2 x = 1.
    Значит cosx ≠ 0 и можно поделить уравнение на cos 2 x. Получим

    tg 2 x – 6 tgx + 8 = 0
    Обозначим tgx = y
    y 2 – 6 y + 8 = 0
    y 1 = 4; y 2 = 2
    а) tgx = 4, x= arctg4 + 2 k , k
    б) tgx = 2, x= arctg2 + 2 k , k .

    Ответ: arctg4 + 2 k , arctg2 + 2 k, k

    4. Уравнения вида a sinx + b cosx = с, с ≠ 0.

    1) Решить уравнение .

    Решение:

    Ответ:

    5. Уравнения, решаемые разложением на множители.

    1) Решить уравнение sin2x – sinx = 0.

    Корнем уравнения f ( х ) = φ ( х ) может служить только число 0. Проверим это:

    cos 0 = 0 + 1 – равенство верно.

    Число 0 единственный корень данного уравнения.

    Ответ: 0.

    Тема: «Методы решения тригонометрических уравнений».

    Цели урока:

    образовательные:

    Сформировать навыки различать виды тригонометрических уравнений;

    Углубление понимания методов решения тригонометрических уравнений;

    воспитательные:

    Воспитание познавательного интереса к учебному процессу;

    Формирование умения анализировать поставленную задачу;

    развивающие:

    Формировать навык проводить анализ ситуации с последующим выбором наиболее рационального выхода из нее.

    Оборудование: плакат с основными тригонометрическими формулами, компьютер, проектор, экран.

    Начнем урок с повторения основного приема решения любого уравнения: сведение его к стандартному виду. Путем преобразований линейные уравнения сводят к виду ах = в, квадратные – к виду ax 2 + bx + c =0. В случае тригонометрических уравнений необходимо свести их к простейшим, вида: sinx = a , cosx = a , tgx = a , которые легко можно решить.

    В первую очередь, конечно, для этого необходимо использовать основные тригонометрические формулы, которые представлены на плакате: формулы сложения, формулы двойного угла, понижения кратности уравнения. Мы уже умеем решать такие уравнения. Повторим некоторые из них:

    Вместе с тем существуют уравнения, решение которых требует знаний некоторых специальных приемов.

    Темой нашего урока является рассмотрение этих приемов и систематизация методов решения тригонометрических уравнений.

    Методы решения тригонометрических уравнений.

    1. Преобразование к квадратному уравнению относительно какой-либо тригонометрической функции с последующей заменой переменной.

    Рассмотрим каждый из перечисленных методов на примерах, но более подробно остановимся на двух последних, так как два первых мы уже использовали при решении уравнений.

    1. Преобразование к квадратному уравнению относительно какой-либо тригонометрической функции.

    2. Решение уравнений методом разложения на множители.

    3. Решение однородных уравнений.

    Однородными уравнениями первой и второй степени называются уравнения вида:

    соответственно (а ≠ 0, b ≠ 0, с ≠ 0).

    При решении однородных уравнений почленно делят обе части уравнения на cosx для (1) уравнения и на cos 2 x для (2). Такое деление возможно, так как sinx и cosx не равны нулю одновременно – они обращаются в нуль в разных точках. Рассмотрим примеры решения однородных уравнений первой и второй степени.

    Запомним это уравнение: при рассмотрении следующего метода – введение вспомогательного аргумента, решим его другим способом.


    4. Введение вспомогательного аргумента.

    Рассмотрим уже решенное предыдущим методом уравнение:

    Как видим, получается тот же результат.

    Рассмотрим еще один пример:

    В рассмотренных примерах было, в общем, понятно, на что требуется разделить исходное уравнение, чтобы ввести вспомогательный аргумент. Но может случиться, что не очевидно, какой делитель выбрать. Для этого существует специальная методика, которую мы сейчас и рассмотрим в общем виде. Пусть дано уравнение.

    Методы решения тригонометрических уравнений.

    Решение тригонометрического уравнения состоит из двух этапов: преобразование уравнения для получения его простейшего вида (см. выше ) и решение полученного простейшего тригонометрического уравнения. Существует семь основных методов решения тригонометрических уравнений.

    1. Алгебраический метод.

    (метод замены переменной и подстановки).

    2. Разложение на множители.

    П р и м е р 1. Решить уравнение: sin x + cos x = 1 .

    Р е ш е н и е. Перенесём все члены уравнения влево:

    Sin x + cos x – 1 = 0 ,

    Преобразуем и разложим на множители выражение в

    Левой части уравнения:

    П р и м е р 2. Решить уравнение: cos 2 x + sin x · cos x = 1.

    Р е ш е н и е. cos 2 x + sin x · cos x – sin 2 x – cos 2 x = 0 ,

    Sin x · cos x – sin 2 x = 0 ,

    Sin x · (cos x – sin x ) = 0 ,

    П р и м е р 3. Решить уравнение: cos 2 x – cos 8 x + cos 6 x = 1.

    Р е ш е н и е. cos 2 x + cos 6 x = 1 + cos 8 x ,

    2 cos 4x cos 2x = 2 cos ² 4x ,

    Cos 4x · (cos 2x – cos 4x ) = 0 ,

    Cos 4x · 2 sin 3x · sin x = 0 ,

    1). cos 4x = 0 , 2). sin 3x = 0 , 3). sin x = 0 ,

    3. Приведение к однородному уравнению.

    Уравнение называется однородным от носительно sin и cos , если все его члены одной и той же степени относительно sin и cos одного и того же угла . Чтобы решить однородное уравнение, надо:

    а ) перенести все его члены в левую часть;

    б ) вынести все общие множители за скобки;

    в ) приравнять все множители и скобки нулю;

    г ) скобки, приравненные нулю, дают однородное уравнение меньшей степени, которое следует разделить на

    cos (или sin ) в старшей степени;

    д ) решить полученное алгебраическое уравнение относительно tan .

    sin 2 x + 4 sin x · cos x + 5 cos 2 x = 2.

    Р е ш е н и е. 3sin 2 x + 4 sin x · cos x + 5 cos 2 x = 2sin 2 x + 2cos 2 x ,

    Sin 2 x + 4 sin x · cos x + 3 cos 2 x = 0 ,

    Tan 2 x + 4 tan x + 3 = 0 , отсюда y 2 + 4y +3 = 0 ,

    Корни этого уравнения: y 1 = — 1, y 2 = — 3, отсюда

    1) tan x = –1, 2) tan x = –3,

    4. Переход к половинному углу.

    Рассмотрим этот метод на примере:

    П р и м е р. Решить уравнение: 3 sin x – 5 cos x = 7.

    Р е ш е н и е. 6 sin (x / 2) · cos (x / 2) – 5 cos ² (x / 2) + 5 sin ² (x / 2) =

    7 sin ² (x / 2) + 7 cos ² (x / 2) ,

    2 sin ² (x / 2) – 6 sin (x / 2) · cos (x / 2) + 12 cos ² (x / 2) = 0 ,

    tan ² (x / 2) – 3 tan (x / 2) + 6 = 0 ,

    . . . . . . . . . .

    5. Введение вспомогательного угла.

    Рассмотрим уравнение вида :

    a sin x + b cos x = c ,

    Где a , b , c – коэффициенты; x – неизвестное.

    Теперь коэффициенты уравнения обладают свойствами синуса и косинуса , а именно : модуль ( абсолютное значение ) каждого из них не больше 1, а сумма их квадратов равна 1 . Тогда можно обозначить их соответственно как cos и sin (здесь — так называемый вспомогательный угол ), и наше уравнение прини

    Метод вспомогательного угла в тригонометрии

    На уроках алгебры учителя рассказывают, что существует небольшой (на самом деле — очень даже большой) класс тригонометрических уравнений, которые не решаются стандартными способами — ни через разложение на множители, ни через замену переменной, ни даже через однородные слагаемые.{2}}=1$, а это значит, что обязательно найдётся такой угол $\alpha $, для которого эти числа являются соответственно косинусом и синусом. Поэтому наше уравнение перепишется следующим образом:

    \[\begin{align}& \cos \alpha \sin x+\sin \alpha \cos x=1 \\& \sin \left( \alpha +x \right)=1 \\\end{align}\]

    А это уже легко решается, после чего останется лишь выяснить, чему равен угол $\alpha $. Как это выяснить, а также как правильно подбирать число для деления обеих частей уравнения (в данном простом примере мы делили на 5) — об этом в сегодняшнем видеоуроке:

    Сегодня мы будем разбирать решение тригонометрических уравнений, а, точнее, один-единственный прием, который называется «метод вспомогательного угла». Почему именно этот метод? Просто потому, что за последние два-три дня, когда я занимался с учениками, которым рассказывал о решении тригонометрических уравнений, и мы разбирали, в том числе, метод вспомогательного угла, и все ученики как один допускают одну и ту же ошибку.{2}}=3+1=4\]

    \[\sqrt{l}=2\]

    $\sqrt{l}$ — это тот самый коэффициент, на который необходимо разделить обе части уравнения, чтобы перед синусом и косинусом появились числа, которые сами по себе являются синусами и косинусами. Давайте разделим:

    \[\frac{\sqrt{3}}{2}\cdot \sin 2x-\frac{1}{2}\cdot \cos 2x=\frac{1}{2}\]

    Посмотрим на то, что у нас получилось слева: существует ли такой $\sin $ и $\cos $, чтобы $\cos \alpha =\frac{\sqrt{3}}{2}$, а $\sin \alpha =\frac{1}{2}$? Очевидно существует: $\alpha =\frac{\text{ }\!\!\pi\!\!\text{ }}{6}$. Поэтому мы можем переписать наше выражение следующим образом:

    \[\cos \frac{\text{ }\!\!\pi\!\!\text{ }}{\text{6}}\cdot \sin 2x-\sin \frac{\text{ }\!\!\pi\!\!\text{ }}{\text{6}}\cdot \cos 2x=\frac{1}{2}\]

    \[\sin 2x\cdot \cos \frac{\text{ }\!\!\pi\!\!\text{ }}{\text{6}}-\cos 2x\cdot \sin \frac{\text{ }\!\!\pi\!\!\text{ }}{\text{6}}=\frac{1}{2}\]

    Теперь перед нами формула синуса разности. Мы можем написать так:

    \[\sin \left( 2x-\frac{\text{ }\!\!\pi\!\!\text{ }}{\text{6}} \right)=\frac{1}{2}\]

    Перед нами простейшая классическая тригонометрическая конструкция. Напомню:

    \[\sin x=a\]

    \[x=\arcsin a+2\text{ }\!\!\pi\!\!\text{ }n\]

    \[x=\text{ }\!\!\pi\!\!\text{ }-\arcsin a-2\text{ }\!\!\pi\!\!\text{ }n\]

    Это и запишем для нашего конкретного выражения:

    \[\left[ \begin{align}& 2x-\frac{\text{ }\!\!\pi\!\!\text{ }}{6}=\frac{\text{ }\!\!\pi\!\!\text{ }}{6}=2\text{ }\!\!\pi\!\!\text{ }n \\& 2x-\frac{\text{ }\!\!\pi\!\!\text{ }}{\text{6}}=\text{ }\!\!\pi\!\!\text{ }-\frac{\text{ }\!\!\pi\!\!\text{ }}{\text{6}}+2\text{ }\!\!\pi\!\!\text{ }n \\\end{align} \right.\]

    \[\left[ \begin{align}& 2x=\frac{\text{ }\!\!\pi\!\!\text{ }}{3}+2\text{ }\!\!\pi\!\!\text{ }n \\& 2x=\text{ }\!\!\pi\!\!\text{ }+2\text{ }\!\!\pi\!\!\text{ }n \\\end{align} \right.\]

    \[\]

    \[\left[ \begin{align}& x=\frac{\text{ }\!\!\pi\!\!\text{ }}{6}+\text{ }\!\!\pi\!\!\text{ }n \\& x=\frac{\text{ }\!\!\pi\!\!\text{ }}{2}+\text{ }\!\!\pi\!\!\text{ }n \\\end{align} \right.\]

    \[n\in Z\]

    Нюансы решения

    Итак, что нужно делать, если вам попалось подобный пример:

    1. Преобразовать конструкцию, если нужно.{2}}=4\]

      \[\sqrt{l}=2\]

      Перепишем с учетом этого факта:

      \[\frac{\sqrt{3}}{2}\cdot \sin 2x-\frac{1}{2}\cdot \cos 2x=\cos x\]

      В этом случае мы можем записать, что $\frac{\sqrt{3}}{2}=\frac{\text{ }\!\!\pi\!\!\text{ }}{3}$, а $\frac{1}{2}=\cos \frac{\text{ }\!\!\pi\!\!\text{ }}{3}$. Перепишем:

      \[\sin \frac{\text{ }\!\!\pi\!\!\text{ }}{\text{3}}\cdot \sin 2x-\cos \frac{\text{ }\!\!\pi\!\!\text{ }}{\text{3}}\cdot \cos 2x=\cos x\]

      \[-\cos \left( \frac{\text{ }\!\!\pi\!\!\text{ }}{\text{3}}+2x \right)=\cos x\]

      Внесем «минус» в скобку хитрым способом. Для этого заметим следующее:

      \[\cos \left( \frac{\text{ }\!\!\pi\!\!\text{ }}{\text{3}}+2x \right)=\cos \left( \text{ }\!\!\pi\!\!\text{ }-\text{ }\!\!\pi\!\!\text{ +}\frac{\text{ }\!\!\pi\!\!\text{ }}{\text{3}}+2x \right)=\]

      \[=\cos \left( \text{ }\!\!\pi\!\!\text{ }-\frac{2\text{ }\!\!\pi\!\!\text{ }}{3}+2x \right)=\cos \left( \text{ }\!\!\pi\!\!\text{ }+\varphi \right)=-\cos \varphi \]

      Возвращаемся к нашему выражению и вспоминаем, что в роли $\varphi $ у нас выражение $-\frac{2\text{ }\!\!\pi\!\!\text{ }}{3}+2x$. Поэтому запишем:

      \[-\left( -\cos \left( -\frac{2\text{ }\!\!\pi\!\!\text{ }}{3}+2x \right) \right)=\cos x\]

      \[\cos \left( 2x-\frac{2\text{ }\!\!\pi\!\!\text{ }}{3} \right)=\cos x\]

      Чтобы решить подобною задачу, нужно вспомнить такое:

      \[\cos \alpha =\cos \beta \]

      \[\left[ \begin{align}& \alpha =\beta +2\text{ }\!\!\pi\!\!\text{ }n \\& \alpha =-\beta +2\text{ }\!\!\pi\!\!\text{ }n \\\end{align} \right.\]

      Разберемся с нашим примером:

      \[\left[ \begin{align}& 2x-\frac{2\text{ }\!\!\pi\!\!\text{ }}{3}=x+2\text{ }\!\!\pi\!\!\text{ }n \\& 2x-\frac{2\text{ }\!\!\pi\!\!\text{ }}{3}=-x+2\text{ }\!\!\pi\!\!\text{ }n \\\end{align} \right.\]

      Давайте посчитаем каждое из этих уравнений:

      \[x=\frac{2\text{ }\!\!\pi\!\!\text{ }}{3}+2\text{ }\!\!\pi\!\!\text{ }n\]

      И вторую:

      \[3x=\frac{2\text{ }\!\!\pi\!\!\text{ }}{3}+2\text{ }\!\!\pi\!\!\text{ }\]

      \[x=\frac{2\pi }{9}+\frac{2\pi n}{3}\]

      Запишем окончательный ответ:

      \[\left[ \begin{align}& x=\frac{2\text{ }\!\!\pi\!\!\text{ }}{3}+2\text{ }\!\!\pi\!\!\text{ }n \\& x=\frac{2\text{ }\!\!\pi\!\!\text{ }}{9}+\frac{2\text{ }\!\!\pi\!\!\text{ }n}{3} \\\end{align} \right.{2}}x}{2}\cdot \sin x=\]

      \[=2\left( 1-\cos 2x \right)\cdot \sin x\]

      А теперь подставим все это в нашу исходную конструкцию:

      \[\sin 2x\cos x+\cos 2x\sin x+2\sin x-2\cos x\sin x+4\cos x=5\]

      \[\sin 2x\cos x-\operatorname{cosx}-cos2\sin x+2\sin x+4\cos x=5\]

      \[\sin \left( 2x-x \right)+2\sin x+4\cos x=5\]

      \[3\sin x+4\cos x=5\]

      Давайте введем нашу поправку:

      \[l=9+16=25\]

      \[\sqrt{l}=5\]

      Записываем:

      \[\frac{3}{5}\sin x+\frac{4}{5}\cos x=1\]

      Таких $\alpha $, для которых $\sin $ или $\cos $ был бы равен $\frac{3}{5}$ и $\frac{4}{5}$ в тригонометрической таблице нет. Поэтому давайте просто так и напишем и сведем выражение к синусу суммы:

      \[\sin x\cdot \cos \varphi +\cos x\cdot \sin \varphi =1\]

      \[\sin \left( x+\varphi \right)=1\]

      Это частный случай, простейшая тригонометрическая конструкция:

      \[x+\varphi =\frac{\text{ }\!\!\pi\!\!\text{ }}{2}+2\text{ }\!\!\pi\!\!\text{ }n\]

      \[x=\frac{\text{ }\!\!\pi\!\!\text{ }}{\text{2}}-\varphi +2\text{ }\!\!\pi\!\!\text{ }n\]

      Осталось найти, чему равен $\varphi $. Именно в этом месте многие ученики ошибаются. Дело в том, что на $\varphi $ накладываются два требования:

      \[\left\{ \begin{align}& \cos \varphi =\frac{3}{5} \\& \sin \varphi =\frac{4}{5} \\\end{align} \right.\]

      Начертим радар и посмотрим, где такие значения встречаются:

      Возвращаясь к нашему выражению, мы напишем следующее:

      \[x=\frac{\text{ }\!\!\pi\!\!\text{ }}{2}-\arcsin \frac{4}{5}+2\text{ }\!\!\pi\!\!\text{ }n\]

      Но и эту запись можно немного оптимизировать. Поскольку мы знаем следующее:

      \[\alpha :\arcsin \alpha +\arccos \alpha =\frac{\text{ }\!\!\pi\!\!\text{ }}{\text{2}},\]

      то в нашем случае можно записать так:

      \[x=\arccos \frac{4}{5}+2\text{ }\!\!\pi\!\!\text{ }n\]

      Пример № 2

      Здесь потребуется еще более глубокое понимание методик решения стандартных задач без тригонометрии. Но для решения этого примера мы также используем метод вспомогательного угла.\[\]

      \[5+2\sin 2x-5\cos x=5\sin x\]

      Первое, что бросается в глаза — здесь нет степеней выше первой и поэтому ничего нельзя разложить по формулам разложения степеней.{2}}=2\]

      \[\sqrt{l}=\sqrt{2}\]

      Разделим все на $\sqrt{2}$:

      \[\left[ \begin{align}& \frac{\sqrt{2}}{2}\sin x+\frac{\sqrt{2}}{2}\cos x=\frac{3}{2\sqrt{2}} \\& \frac{\sqrt{2}}{2}\sin x+\frac{\sqrt{2}}{2}\cos x=\frac{\sqrt{2}}{2} \\\end{align} \right.\]

      Все сведем к $\cos $:

      \[\cos x\cdot \cos \frac{\text{ }\!\!\pi\!\!\text{ }}{4}+\sin x\sin \frac{\text{ }\!\!\pi\!\!\text{ }}{\text{4}}\]

      \[\left[ \begin{align}& \cos \left( x-\frac{\text{ }\!\!\pi\!\!\text{ }}{\text{4}} \right)=\frac{3}{2\sqrt{2}} \\& \cos \left( x-\frac{\text{ }\!\!\pi\!\!\text{ }}{4} \right)=\frac{\sqrt{2}}{2} \\\end{align} \right.\]

      Разбираемся с каждым из этих выражений.

      Первое уравнение корней не имеет, и для доказательства этого факта нам поможет иррациональность в знаменателе. Заметим следующее:

      \[\sqrt{2}<1,5\]

      \[\frac{3}{2\sqrt{2}}>\frac{3}{3\cdot 1,5}=\frac{3}{3}=1\]

      Итого мы четко доказали, что требуется, чтобы $\cos \left( x-\frac{\text{ }\!\!\pi\!\!\text{ }}{4} \right)$ был равен числу, которое большее «единицы» и, следовательно, у этой конструкции корней нет.

      Разбираемся со вторым:

      \[x-\frac{\text{ }\!\!\pi\!\!\text{ }}{4}=\pm \frac{\text{ }\!\!\pi\!\!\text{ }}{4}+2\text{ }\!\!\pi\!\!\text{ }n\]

      Решаем эту конструкцию:

      \[x=\frac{\text{ }\!\!\pi\!\!\text{ }}{4}\pm \frac{\text{ }\!\!\pi\!\!\text{ }}{\text{4}}+2\text{ }\!\!\pi\!\!\text{ }n\]

      В принципе, можно оставить ответ таким, а можно его расписать:

      \[x=\frac{\text{ }\!\!\pi\!\!\text{ }}{2}+2\text{ }\!\!\pi\!\!\text{ }n\]

      \[x=2\text{ }\!\!\pi\!\!\text{ }n\]

      \[n\in Z\]

      Важные моменты

      В заключение хотел бы еще раз обратить ваше внимание на работу с «некрасивыми» аргументами, т.е. когда $\sin $ и $\cos $ не являются табличными значениями. Проблема состоит в том, что если мы утверждаем, что в нашем уравнении $\frac{3}{5}$ — это $\cos $, а $\frac{4}{5}$ — это $\sin $, то в итоге, после того как мы решим конструкцию, нужно учитывать оба этих требования. Мы получаем систему из двух уравнений. Если мы не будем это учитывать, то получим следующую ситуацию. В этом случае мы получим две точки и на месте $\varphi $ у нас окажется два числа: $\arcsin \frac{4}{5}$ и $-\arcsin \frac{4}{5}$, однако последний нас ни в коем случае не устраивает. То же самое будет и с точкой $\frac{3}{5}$.

      Такая проблема возникает только тогда, когда речь идет о «некрасивых» аргументах. Когда у нас табличные значения, то ничего такого нет.

      Надеюсь, сегодняшний урок помог вам разобраться, что такое метод вспомогательного угла и как его применять на примерах разного уровня сложности. Но это не единственный урок, посвященный решению задач методом вспомогательного угла. Поэтому оставайтесь с нами!

      Смотрите также:

      1. Как решать тригонометрические уравнения? Основные приёмы и методы.
      2. Решаем однородные тригонометрические уравнения
      3. Решение задач B12: №448—455
      4. Пробный ЕГЭ-2011 по математике, вариант №4
      5. Задача C2: уравнение плоскости через определитель
      6. Тест по задачам B14: средний уровень, 2 вариант

      Тригонометрических уравнений и их решений — Учебный материал для IIT JEE

      Тригонометрическое уравнение

      Уравнение, включающее одно или несколько тригонометрических отношений неизвестных углов, называется тригонометрическим уравнением. Тригонометрическое уравнение можно записать как Q 1 (sin θ, cos θ, tan θ, cot θ, sec θ, cosec θ) = Q 2 (sin θ, cos θ, tan θ, cot θ, sec θ , cosec θ), где Q 1 и Q 2 — рациональные функции.

      Пример: Рассмотрим уравнение cos 2 x — 4 sin x = 1.

      Это тригонометрическое уравнение, а не тождество, поскольку оно не выполняется для всех значений x, например. уравнение не выполняется при (2n + 1) π / 4.

      Решение тригонометрического уравнения:

      Все возможные значения неизвестного, которые удовлетворяют данному уравнению, называются решением данного уравнения.

      Для полного решения должны быть получены «все возможные значения», удовлетворяющие уравнению.

      Когда мы пытаемся решить тригонометрическое уравнение, мы пытаемся найти все наборы значений θ, которые удовлетворяют данному уравнению.Иногда в простых уравнениях и когда легко нарисовать график уравнения, можно найти решение, просто просмотрев график.

      Период функции:

      Функция f (x) называется периодической, если существует T> 0 такое, что f (x + T) = f (x) для всех x в области определения f (x). Если T — наименьшее положительное действительное число такое, что f (x + T) = f (x), то оно называется периодом f (x).

      Тригонометрические функции, такие как sin, cos и tan, являются периодическими функциями.

      Иллюстрация: Мы пытаемся найти решения уравнения sin θ = 0, отличные от θ = 0. Увидев уравнение, можно сразу прийти к выводу, что θ = 0 — единственное решение. Но в случае тригонометрических уравнений важно исключить все возможности, чтобы найти правильное решение.

      Пусть OX будет начальной строкой

      Пусть ∠POX = θ и OP = r

      от ΔPOL,

      sin θ = PL / OP = y / r.

      Теперь sin θ = 0

      ⇒ y / r = 0; ⇒ y = 0.

      Это возможно только тогда, когда OP совпадает с OX или OX ’.

      Когда OP совпадает с OX, θ = 0, ± 2π, ± 4π и ± 6π ……… (1)

      И когда OP совпадает с OX ’, θ = ± π, ± 3π, ± 5π ……… (2)

      Таким образом, из (1) и (2) следует, что при sin θ = 0

      θ = nπ, где n = 0, ± 1, ± 2, ………

      Мы называем θ = nπ общим решением тригонометрического уравнения sin θ = 0, потому что для всех значений n это решение удовлетворяет данному уравнению.

      Иллюстрация: Общее решение cos θ = 0

      cos θ = 0 ⇒ x = π / 2.

      Это возможно только тогда, когда OP совпадает с OY или OY ’

      Когда OP совпадает с OY,

      θ = π / 2, 5π / 2, 9π / 2 или, -3π / 2, -7π / 2 .. ……… (1)

      , когда OP совпадает с OY ’

      θ = -3π / 2, -7π / 2 или, -π / 2, -5π / 2 ………… (2)

      Таким образом, из (1) и (2) следует, что общее решение cos θ = 0 есть θ (2n + 1) π / 2, где n = 0, ± 1, ± 2 ………

      Подробнее о тригонометрических уравнениях см. В видео ниже:

      Общее решение уравнения sin θ = k.

      Мы знаем, что когда sin θ = k, k должно быть таким, что –1 ≤ k ≤ 1

      Всегда можно найти α ∈ [–π / 2, π / 2]

      Так как sin (-π) / 2 = -1 & sin π / 2 = 1, так что sin θ = k, т.е. α = sin-1k

      , т.е. sin θ = sin α, α ∈ [–π / 2, π / 2]

      ⇒ грех θ — грех α = 0

      ⇒ 2 sin {(θ — α) / 2} cos {θ + α) / 2} = 0

      из приведенного выше уравнения, либо sin {(θ — α) / 2) = 0

      и, следовательно, ((θ — α) / 2) = целое кратное π

      ∴ θ — α = 2nπ

      и.е. θ = 2nπ + α

      θ = 2nπ + (–1) 2n α, где n = 0, ± 1, ± 2… (1)

      или, cos {(θ + α) / 2} = 0

      , т.е. {(θ + α) / 2} = любое нечетное кратное π / 2

      , т.е. {(θ + α) / 2} = (2n + 1) π / 2

      , т.е. θ = (2n + 1) π — α

      ⇒ θ = (2n +1) π + (–1) 2n + 1 α… (2)

      Из (1) и (2) заключаем, что

      θ = nπ + (–1) n α, где n — целое кратное, является общим решением уравнения sin θ = k

      Тригонометрические уравнения и их общие решения:

      Тригонометрическое уравнение

      Общее решение

      грех θ = 0

      Тогда θ = nπ

      cos θ = 0

      θ = (nπ + π / 2)

      тангенс угла θ = 0

      θ = nπ

      грех θ = 1

      θ = (2nπ + π / 2) = (4n + 1) π / 2

      cos θ = 1

      θ = 2nπ

      грех θ = грех α

      θ = nπ + (-1) nα, где α ∈ [-π / 2, π / 2]

      cos θ = cos α

      θ = 2nπ ± α, где α ∈ (0, π]

      тангенс угла θ = тангенс угла α

      θ = nπ + α, где α ∈ (-π / 2, π / 2]

      sin2 θ = sin2 α

      θ = nπ ± α

      cos2 θ = cos2 α

      θ = nπ ± α

      tan2 θ = tan2 α

      θ = nπ ± α

      Если предполагается, что α является наименьшим положительным значением θ, которое удовлетворяет двум данным тригонометрическим уравнениям, то общее значение θ будет 2nπ + α.

      Иллюстрация: Найдите общее решение уравнения sin θ = 1/2

      Решение: Мы знаем, что sin θ = 1/2 = sin π / 6.

      Итак, общее решение данного уравнения: θ = nπ + (–1) nπ / 6, n ∈ 0, ± 1, ± 2

      Иллюстрация: Решите уравнение sin 6x + sin 4x = 0.

      Решение: Применяя формулы для суммы синусов, т.е.

      грех А + грех В = грех (А + В) / 2. cos (A-B) / 2, имеем

      sin 5x cos x = 0 ……… (1)

      Если «x» является решением уравнения, то верно хотя бы одно из следующих уравнений:

      sin 5x = 0 или cos x = 0 ……… (2)

      И наоборот, если x является решением одного из уравнений (2), то это также решение уравнения (1).Таким образом, уравнение (1) эквивалентно уравнению (2). Решения уравнения (2) даются формулой

      x = nπ / 5, x = (2n + 1) π / 2, где n = 0, ± 1, ± 2 ……

      Все эти значения x и только эти значения являются решениями исходного уравнения.

      Иллюстрация: Найдите наиболее общее значение θ, где sin θ = — √3 / 2 и tan θ = √3.

      Решение: Знаки имеют большое значение в случае тригонометрических функций. Студенты обычно склонны упоминать общие решения sin и tan θ, что неверно, поскольку это не дает нам полного решения.

      sin θ отрицателен в 3-м и 4-м квадрантах, а tan θ положителен в 1-м и 3-м квадрантах.

      Таким образом, обычным является 3-й квадрант, и при θ = 4π / 3 удовлетворяются оба.

      ∴ Общее решение — 2nπ + 4π / 3.

      Это потому, что в интервале [0, 2π] выполняется только при 4π / 3. Снова в [2π, 4π] это выполняется при 2π + 4π / 3 и так далее.

      Следовательно, общее решение уравнения равно 2nπ + 4π / 3.

      Иллюстрация: Найдите общее решение для cos 3θ = sin 2θ.

      Решение: Эту проблему можно решить двумя способами.

      Метод 1: Мы можем записать данное уравнение как

      cos 3θ = cos (π / 2 — 2θ)

      ⇒ 3θ = 2nπ + (π / 2 — 2θ), где n = 0, ± 1, ± 2 ……

      или 5θ = 2nπ + π / 2, а также θ = 2nπ — π / 2

      или θ = (4n + 1) π / 10 и

      θ = (4n – 1) π / 2, где n ∈ I …… (A)

      Метод 2: sin 2θ = sin (π / 2 — 3θ)

      2θ = nπ + (–1) n (π / 2 — 3θ).

      Случай I: Когда n четное, n = 2m, где m = 0, ± 1, ± 2 ……

      2θ = 2mπ + π / 2 — 3θ

      θ = (4m + 1) π / 10, где m ∈ I ……. (В)

      Случай II: Если n нечетное, n = (2m + 1)

      2θ = (2m + 1) π — (π / 2 — 3θ)

      θ = — (4m + 1) π / 2, где m = 0, ± 1, ± 2 …… (B)

      Примечание: Без сомнения, решения, полученные обоими методами для нечетных значений n, различны, но, как показано на диаграмме ниже, вы можете видеть, что все возможные значения θ могут быть получены обоими данными решениями:

      из B

      от A

      для m = 0, θ = — π / 2,

      для n = 0, θ = — π / 2

      для m = 1, θ = — 5π / 2,

      для n = 1, θ = + 3π / 2

      для m = 2, θ = — 9π / 2,

      для n = 2, θ = + 7π / 2

      для m = –1, θ = — 3π / 2,

      для m = –1, θ = — 5π / 2

      для m = –2, θ = 7π / 2

      для m = –2, θ = — 9π / 2

      Общее решение sin 2 θ = k, где k ∈ [0, 1]

      Учитывая, что sin 2 θ = k, k ∈ [0, 1]

      Мы можем найти такое α, что

      ⇒ sin 2 θ = sin 2 α, где α = sin -1 √k

      и.е. (грех θ — грех α) (грех θ + грех α) = 0

      либо sin θ — sin α = 0

      θ = nπ + (–1) n α, где n = 0, ± 1, ± 2 ……… (1)

      или, sin θ + sin α = 0

      sin θ = — sin α

      θ = nπ — (–1) n α, где n = 0, ± 1, ± 2 …… .. (2)

      Из (1) и (2) получаем общее решение уравнения для данного

      θ = nπ ± α, где n = 0, ± 1, ± 2 …… и α = sin -1 √k

      Иллюстрация: Решите уравнение 7tan 2 θ — 9 = 3 секунды 2 θ

      Решение: Дано, 7tan 2 θ — 9 = 3 секунды 2 θ

      или, 7tan 2 θ — 9 = 3 (1 + tan 2 θ)
      или, 4tan 2 θ = 12
      или, tan 2 θ = 3
      или, tan 2 θ = (tan π / 3) 2
      ⇒ θ = nπ + π / 3, где n = 0, ± 1, ± 2 …………

      Примечание : Мы не можем определить уникальный метод решения тригонометрических уравнений.В каждом случае успех решения тригонометрического уравнения зависит, в частности, от знания и умения применять тригонометрические формулы, а также от практики решения задач. Многие тригонометрические формулы являются истинными равенствами для всех значений переменных, входящих в них.

      Иллюстрация: Решите уравнение: cos θ = 0
      Решение: Мы можем решить его, чтобы получить две формы
      cos θ = 0 ⇒ θ = (2n + 1) π / 2

      cos θ = cos π / 2 ⇒ θ = 2nπ + π / 2
      или θ = (4n + 1) π / 2.

      Важно: Следующие советы и шаги помогут вам систематически решать тригонометрические уравнения.

      1. Попытайтесь сократить уравнение в терминах одного единственного тригонометрического отношения, предпочтительно sin θ или cos θ.
      Если у нас есть выбор преобразовать задачу в синус или косинус, тогда форма косинуса удобнее по сравнению с формой синуса. Это связано с тем, что в общем решении синуса нам придется иметь дело с (–1) n , что неудобно по сравнению с рассмотрением +, полученного в форме косинуса.
      2. Факторизуйте полином в терминах этих отношений.
      3. Чтобы LHS был равен нулю, решите для каждого фактора. И запишите общее решение для каждого фактора на основе стандартных результатов, полученных ранее в этом разделе.

      например sin θ — k 1 = 0 ⇒ θ = nπ + (–1) n sin -1 k 1

      cos θ — k 2 = 0 ⇒ θ = 2nπ + cos –1 k 2 .

      Внимание: Вы должны проверить, что k 1 , k 2 ∈ [–1, 1].Не пишите вслепую, как есть, иначе будет абсурдно, если они не принадлежат [–1, 1].

      Иллюстрация: Решите уравнение 5sin θ — 2 cos 2 θ — 1 = 0

      Решение: Дано, 5 sin θ — 2 cos 2 θ — 1 = 0

      или, 5 sin θ — 2 (1 — sin 2 θ) — 1 = 0
      или, 2 sin 2 θ + 5 sin θ — 3 = 0
      или, (sin θ + 3) (2 sin θ — 1) = 0
      ∴ sin θ = -3 или sin θ = ½

      Сначала рассмотрим случай, когда sin θ = -3.

      Но этот случай невозможен, так как диапазон синуса составляет [-1, 1].

      Когда sin θ = ½

      Тогда sin θ = sin π / 6.

      ⇒ θ = nπ + (–1) n π / 6, где n = 0, ± 1, ± 2 ………

      Примечание: Никогда не делите на любое нулевое выражение. например Если данное уравнение (sin θ — cos θ) (A) = (B), где A и B обозначают тригонометрические уравнения, то вы можете разделить на (sin θ — cos θ) только тогда, когда θ ≠ nπ + π / 4

      Иллюстрация: Решите уравнение tan θ + sec θ = √3

      Решение: тангенс угла θ + сек θ = √3 …… (1)

      Тогда (sin θ) / (cos θ) + 1 / (cos θ) = √3 …… (2)

      или, cos (θ + π / 6) = cos π / 3

      Общее решение θ + π / 6 = 2nπ ± π / 3, n ∈ I

      Принимая положительный знак, θ + π / 6 = 2nπ + π / 3

      ⇒ θ = 2nπ + π / 6

      Принимая отрицательный знак, θ + π / 6 = 2nπ — π / 3

      ⇒ θ = 2nπ — π / 2

      и.е. θ = (4n — 1) π / 2.

      Но полученное решение является правильным, только если, cos θ ≠ 0, иначе (2) не определено.

      то есть θ ≠ нечетное кратное π / 2

      ⇒ θ ≠ (4n — 1) π / 2.

      Следовательно, общее решение будет θ = 2nπ + π / 6 только тогда, когда n = 0, ± 1, ± 2 ……

      Примечание: Домен уравнения не должен изменяться. В случае его изменения необходимо внести необходимые изменения в общее решение.

      Иллюстрация: Решите уравнение tan 5θ = tan 3θ

      Решение: Теперь tan 5θ = tan 3θ

      ⇒ 5θ = nπ + 3θ

      или, 2θ = nπ

      θ = nπ / 2, где n = 0, ± 1, ± 2 ……

      положить n = 0 дает θ = 0, исходное уравнение выполнено

      положив n = 1, получаем θ = π / 2, уравнение принимает вид tan 5π / 2 = tan 3π / 2.

      Уравнение не определено для нечетного числа, кратного π / 2.

      Отсюда заключаем, что θ = 2nπ, где n = 0, ± 1, ± 2 ………

      Некоторые ключевые моменты, на которые следует обратить внимание:

      1. Если в уравнении участвует tan θ или sec θ, θ ≠ нечетное число, кратное π / 2.

      2. Если в уравнении участвует cot θ или cosec θ, θ кратно π или 0.

      Тригонометрия полна формул, и студентам рекомендуется изучить все тригонометрические формулы, включая основы тригонометрии, чтобы оставаться конкурентоспособными на экзаменах JEE и других инженерных экзаменах.Студенты должны практиковать различные задачи тригонометрии, основанные на тригонометрических соотношениях и основах тригонометрии, чтобы познакомиться с темой.

      Вы можете сослаться на некоторые из связанных ресурсов, перечисленных ниже:

      Чтобы узнать больше, купите учебные материалы по Тригонометрия , включающие учебные заметки, заметки о пересмотре, видеолекции, решенные вопросы за предыдущий год и т. Д. Также дополнительные учебные материалы по математике можно найти здесь .

      Основные тригонометрические уравнения

      Углы (аргументы функций): \ (x, \) \ ({x_1}, \) \ ({x_2} \)
      Набор целых чисел: \ (\ mathbb {Z} \)
      Целое число: \ (n \)
      Вещественное число: \ (a \)

      Тригонометрические функции: \ (\ sin x, \) \ (\ cos x, \) \ (\ tan x, \) \ (\ cot x \)
      Обратные тригонометрические функции: \ (\ arcsin a, \) \ (\ arccos a, \) \ (\ arctan a, \) \ (\ text {arccot} a \)

      1. Уравнение, включающее тригонометрические функции от неизвестного угла, называется тригонометрическим уравнением.n} \ arcsin a + \ pi n, \) \ (n \ in \ mathbb {Z}. \)
        Эта формула содержит две ветви решений:
        \ ({x_1} = \ arcsin a + 2 \ pi n \ ), \ ({X_2} = \ pi — \ arcsin a + 2 \ pi n, \) \ (n \ in \ mathbb {Z} \).
      2. В простом случае \ (\ sin x = 1 \) решение имеет вид
        \ (x = \ pi / 2 + 2 \ pi n, \) \ (n \ in \ mathbb {Z} \).
      3. Аналогично, решение уравнения \ (\ sin x = -1 \) дается формулой
        \ (x = — \ pi / 2 + 2 \ pi n, \) \ (n \ in \ mathbb {Z} \ ).
      4. Случай \ (\ sin x = 0 \) (нули синуса)
        \ (x = \ pi n, \) \ (n \ in \ mathbb {Z} \).
      5. Уравнение \ (\ cos x = a \)

      6. Если \ (\ left | a \ right | \ gt 1, \), уравнение \ (\ cos x = a \) не имеет решений.
      7. Если \ (\ left | a \ right | \ le 1, \), общее решение уравнения \ (\ cos x = a \) имеет вид
        \ (x = \ pm \ arccos a + 2 \ pi n , \) \ (n \ in \ mathbb {Z}. \)
        Эта формула включает два набора решений:
        \ ({x_1} = \ arccos a + 2 \ pi n \), \ ({x_2} = — \ arccos a + 2 \ pi n, \) \ (n \ in \ mathbb {Z} \).
      8. В случае \ (\ cos x = 1 \) решение записывается как
        \ (x = 2 \ pi n, \) \ (n \ in \ mathbb {Z} \).
      9. Случай \ (\ cos x = -1 \)
        \ (x = \ pi + 2 \ pi n, \) \ (n \ in \ mathbb {Z} \).
      10. Случай \ (\ cos x = 0 \) (нули косинуса)
        \ (x = \ pi / 2 + \ pi n, \) \ (n \ in \ mathbb {Z} \).
      11. Уравнение \ (\ tan x = a \)

      12. Для любого значения \ (a \) общее решение уравнения \ (\ tan x = a \) имеет вид
        \ (x = \ arctan a + \ pi n, \) \ (n \ in \ mathbb {Z}. \)
      13. Случай \ (\ tan x = 0 \) (нули касательной)
        \ (x = \ pi n, \) \ (n \ in \ mathbb {Z}. \)
      14. Уравнение \ (\ cot x = 0 \)

      15. Для любого значения \ (a \) общее решение тригонометрического уравнения \ (\ cot x = 0 \) записывается как
        \ (x = \ text {arccot} a + \ pi n, \) \ ( п \ в \ mathbb {Z}.\)
      16. Случай \ (\ cot x = 0 \) (нули котангенса)
        \ (x = \ pi / 2 + \ pi n, \) \ (n \ in \ mathbb {Z}. \)

      Как найти общее решение тригонометрических уравнений?

      Как найти общее решение тригонометрических уравнений?

      Тригонометрические уравнения

      Определение:
      Уравнение, включающее одно или несколько тригонометрических соотношений неизвестного угла, называется тригонометрическим уравнением

      Тригонометрическое уравнение отличается от тригонометрических тождеств.Идентичность выполняется для каждого значения неизвестного угла , например ., Cos 2 x = 1 — sin 2 x истинно ∀ x ∈ R, в то время как тригонометрическое уравнение выполняется для некоторых конкретных значений неизвестного угла .

      (1) Корни тригонометрического уравнения: Значение неизвестного угла (переменная величина), которое удовлетворяет данному уравнению, называется корнем уравнения, например, ., Cos θ = ½, корень равен θ = 60 ° или θ = 300 °, потому что уравнение выполняется, если мы положим θ = 60 ° или θ = 300 °.

      (2) Решение тригонометрических уравнений: Значение неизвестного угла, удовлетворяющее тригонометрическому уравнению, называется его решением.
      Поскольку все тригонометрические отношения периодичны по своей природе, обычно тригонометрическое уравнение имеет более одного решения или бесконечное число решений. В основном есть три типа решений:

      1. Частное решение: Определенное значение неизвестного угла, удовлетворяющее уравнению.
      2. Главное решение: Наименьшее числовое значение неизвестного угла, удовлетворяющее уравнению (Наименьшее числовое частное решение).
      3. Общее решение: Полный набор значений неизвестного угла, удовлетворяющий уравнению. Он содержит все частные решения, а также основные решения.

      Тригонометрические уравнения с их общим решением

      Тригонометрическое уравнение Общее решение
      грех θ = 0 θ = nπ
      cos θ = 0 θ = nπ + π / 2
      тангенс угла θ = 0 θ = nπ
      грех θ = 1 θ = 2nπ + π / 2
      cos θ = 1 θ = 2nπ
      грех θ = грех α θ = nπ + (−1) n α
      cos θ = cos α θ = 2nπ ± α
      tan θ = tan α θ = nπ ± α
      sin 2 θ = sin 2 α θ = nπ ± α
      tan 2 θ = tan 2 α θ = nπ ± α
      cos 2 θ = cos 2 α θ = nπ ± α
      sin θ = sin α
      cos θ = cos α
      θ = nπ + α
      sin θ = sin α
      tan θ = tan α
      θ = nπ + α
      tan θ = tan α
      cos θ = cos α
      θ = nπ + α

      Общее решение вида a cos θ + b sin θ = c

      Метод определения главного значения

      Предположим, нам нужно найти главное значение sin θ = −½, удовлетворяющее уравнению.
      Поскольку sin θ отрицателен, θ будет в квадранте 3 rd или 4 th . Мы можем подойти к 3-му или 4-му квадранту с двух сторон. Если мы возьмем направление против часовой стрелки, числовое значение угла будет больше π. Если подойти к нему по часовой стрелке, угол будет численно меньше π. За главное значение мы должны взять численно наименьший угол. Итак, для главного значения.
      (1) Если угол находится в 1-м или 2-м квадранте, мы должны выбрать направление против часовой стрелки, а если угол находится в квадранте 3 rd или 4 th , мы должны выбрать направление по часовой стрелке.
      (2) Главное значение никогда не может быть численно больше π.
      (3) Главное значение всегда находится в первом круге (т.е. в первом повороте). По вышеуказанным критериям θ будет -π / 6 или -5π / 6. Между этими двумя -π / 6 имеет наименьшее числовое значение. Следовательно, −π / 6 — это главное значение θ, удовлетворяющее уравнению sin θ = −½.
      Из приведенного выше обсуждения метод нахождения главного значения можно резюмировать следующим образом:

      1. Сначала нарисуйте тригонометрический круг и отметьте квадрант, в котором может лежать угол.
      2. Выберите направление против часовой стрелки для квадрантов 1 и 2 и выберите направление по часовой стрелке для квадрантов 3 и 4 .
      3. Найдите угол при первом повороте.
      4. Выберите численно наименьший угол. Найденный таким образом угол будет главной величиной.
      5. В случае, если два угла, один с положительным знаком, а другой с отрицательным знаком, соответствуют численно наименьшему углу, тогда принято выбирать угол с положительным знаком в качестве главного значения.

      Тригонометрические уравнения с решениями

      1.

      Решение:

      2.

      Решение:

      3.

      Решение:

      4.
      5.


      Решение:

      6.

      Решение:

      7.

      Решение:

      8.

      Решение:

      9.

      Решение:

      Основные тригонометрические уравнения :

      Когда просят решить 2x — 1 = 0, мы можем легко получить 2x = 1 и x = в качестве ответа.
      Когда просят решить 2 sin x — 1 = 0, мы действуем аналогичным образом. Сначала мы смотрим на sin x как на переменную уравнения и решаем, как в первом примере.
      2 sin x — 1 = 0
      2 sin x = 1
      sin x = 1/2

      Знаки и квадранты :

      Решения тригонометрических уравнений также можно найти, исследуя знак значения триггера и определяя соответствующий квадрант (квадранты) для этого значения.

      Сводка тригонометрических формул

      Сводка тригонометрических формул

      Эти формулы относятся к длине и площади определенных кругов или треугольников. На следующей странице вы найдете личности. Идентичности не относятся к конкретным геометрическим фигурам, но верны для всех углов.

      Формулы дуг и секторов окружностей

      Вы можете легко найти как длину дуги, так и площадь сектора для угла θ в окружности радиуса r .

      Длина дуги. Длина дуги равна радиусу r , умноженному на угол θ , где угол измеряется в радианах. Чтобы преобразовать градусы в радианы, умножьте количество градусов на π /180.
      Площадь сектора. Площадь сектора равна половине квадрата радиуса, умноженного на угол, где, опять же, угол измеряется в радианах.
      Формулы для прямоугольных треугольников

      Наиболее важные формулы для тригонометрии — формулы для прямоугольного треугольника.Если θ — один из острых углов в треугольнике, то синус тета — это отношение противоположной стороны к гипотенузе, косинус — это отношение соседней стороны к гипотенузе, а тангенс — это отношение сторона, противоположная соседней стороне.

      Эти три формулы известны мнемоническим языком SohCahToa. Помимо этого, существует очень важная формула Пифагора, которая гласит, что квадрат гипотенузы равен сумме квадратов двух других сторон.

      Зная, что два острых угла дополняют друг друга, то есть они складываются в 90 °, вы можете решить любой прямоугольный треугольник:

      • Если вы знаете две из трех сторон, вы можете найти третью сторону и оба острых угла.
      • Если вы знаете один острый угол и одну из трех сторон, вы можете найти другой острый угол и две другие стороны.
      Формулы наклонных треугольников

      Эти формулы работают для любого треугольника, будь то острый, тупой или прямой.Мы будем использовать стандартную запись, в которой три вершины треугольника обозначаются прописными буквами A, , B и C , а три противоположные им стороны соответственно обозначаются строчными буквами a , . b и c .

      Есть две важные формулы для наклонных треугольников. Их называют законом косинусов и законом синусов.

      Закон косинусов обобщает формулу Пифагора на все треугольники.В нем говорится, что c 2 , квадрат одной стороны треугольника, равен a 2 + b 2 , сумме квадратов двух других сторон минус 2. ab cos & nbsp C , удвоить их произведение, умноженное на косинус противоположного угла. Когда угол C правильный, он становится формулой Пифагора.

      Закон синусов гласит, что отношение синуса одного угла к противоположной стороне является одинаковым для всех трех углов.

      С помощью этих двух формул вы можете решить любой треугольник:

      • Если вы знаете два угла и сторону, вы можете найти третий угол и две другие стороны.
      • Если вы знаете две стороны и включенный угол, вы можете найти третью сторону и оба других угла.
      • Если вы знаете две стороны и угол, противоположный одной из них, есть две возможности для угла, противоположного другой (острый и тупой), и для обеих возможностей вы можете определить оставшийся угол и оставшуюся сторону.
      Формулы площади для треугольников

      Существует три различных полезных формулы для определения площади треугольника, и какая из них вы используете, зависит от того, какая информация у вас есть.

      Половина основания, умноженная на высоту. Это обычный вариант, поскольку он самый простой и обычно у вас есть такая информация. Выбирайте любую сторону для звонка на базу b . Тогда, если h — это расстояние от противоположной вершины до b , то площадь равна половине bh .
      Формула Герона. Это полезно, если вы знаете три стороны треугольника: a , b и c , и все, что вам нужно знать, — это площадь. Пусть s будет половиной их суммы, называемой полупериметром . Тогда площадь является квадратным корнем из произведения s , s a , s b и s c .
      Формула стороны-угла-стороны. Используйте это, если вам известны две стороны, a и b , и включенный угол C . Площадь равна половине произведения двух сторон, умноженного на синус включенного угла.

      Решение простых (и средней сложности) триггерных уравнений

      Purplemath

      При решении тригонометрических уравнений используются как исходные углы, так и тригонометрические тождества, которые вы запомнили, а также большая часть изученной вами алгебры.Будьте готовы к тому, что для решения этих уравнений потребуется думать .

      Далее предполагается, что вы хорошо разбираетесь в значениях триггерного отношения в первом квадранте, как работает единичный круг, соотношение между радианами и градусами и как выглядят кривые различных триггерных функций, на минимум по первому периоду. Если вы не уверены в себе, вернитесь и сначала просмотрите эти темы.


      MathHelp.com

      • Решить sin (
        x ) + 2 = 3 в интервале 0 ° & leq; x <360 °

      Как и в случае с линейными уравнениями, я сначала выделю член, содержащий переменную:

      грех ( x ) + 2 = 3

      грех ( x ) = 1

      Теперь я воспользуюсь запомненными углами отсчета, чтобы получить окончательный ответ.

      Примечание. В инструкциях указан интервал в градусах, что означает, что я должен давать свой ответ в градусах. Да, синус в первом периоде принимает значение 1 при

      π / 2 радиан, но это не тот тип угловой меры, который им нужен, и использование этого в качестве моего ответа, вероятно, приведет к моему как минимум проигрышу. несколько моментов по этому вопросу.

      Итак, в градусах мой ответ:


      • Решить tan
        2 (θ) + 3 = 0 на интервале 0 ° & leq; θ <360 °

      Есть соблазн быстро вспомнить, что тангенс 60 ° включает в себя квадратный корень из 3, и отбросить ответ, но это уравнение на самом деле не имеет решения.Я вижу это, когда замедляюсь и делаю шаги. Мой первый шаг:

      Может ли любой квадрат (касательной или любой другой триггерной функции) быть отрицательным ? Нет! Итак, мой ответ:


      • Решить в интервале 0 ° & leq;
        x <360 °

      Левая часть этого уравнения множится.Я привык делать простой факторинг, например:

      2 y 2 + 3 y = 0

      y (2 y + 3) = 0

      … и затем решить каждый из факторов. То же самое и здесь. Чтобы решить уравнение, которое они мне дали, я начну с факторинга:

      Я занимался алгеброй; то есть, я произвел факторинг, а затем решил каждое из двух уравнений, связанных с факторами.Это создало два тригонометрических уравнения. Итак, теперь я могу сделать триггер; а именно решение этих двух результирующих тригонометрических уравнений, используя то, что я запомнил о косинусоиде. Из первого уравнения я получаю:

      Из второго уравнения я получаю:

      Соединяя эти два набора решений вместе, я получаю решение для исходного уравнения как:

      x = 30 °, 90 °, 270 °, 330 °


      • Решить sin
        2 (θ) — sin (θ) = 2 на интервале 0 & leq; θ <2π

      Во-первых, перенесу все по одну сторону от знака «равно»:

      sin 2 (θ) — sin (θ) — 2 = 0

      Это уравнение является «квадратичным по синусу»; то есть форма уравнения — это формат квадратного уравнения:

      В случае уравнения, которое они хотят, чтобы я решил, X = sin (θ), a = 1, b = –1 и c = –2.

      Поскольку это квадратичная форма, я могу применить некоторые методы квадратного уравнения. В случае этого уравнения я могу разложить на множители квадратичный:

      sin 2 (θ) — sin (θ) — 2 = 0

      (грех (θ) — 2) (грех (θ) + 1) = 0

      Первый фактор дает мне соответствующее тригонометрическое уравнение:

      Но синус никогда не бывает больше 1, поэтому это уравнение не разрешимо; у него нет решения.

      Другой фактор дает мне второе связанное тригонометрическое уравнение:

      грех (θ) + 1 = 0

      sin (θ) = –1

      θ = (3/2) π

      Тогда мой ответ:

      (Если вы выполняете решения только для степеней в своем классе, указанное выше значение решения равно «270 °».)


      • Решите cos
        2 (α) + cos (α) = sin 2 (α) на интервале 0 ° & leq; x <360 °

      Я могу использовать триггерное тождество, чтобы получить квадратичный косинус:

      cos 2 (α) + cos (α) = sin 2 (α)

      cos 2 (α) + cos (α) = 1 — cos 2 (α)

      2cos 2 (α) + cos (α) — 1 = 0

      (2cos (α) — 1) (cos (α) + 1) = 0

      cos (α) = 1/2, cos (α) = –1

      Первое тригонометрическое уравнение, cos (α) = 1/2, дает мне α = 60 ° и α = 300 °.Второе уравнение дает мне α = 180 °. Итак, мое полное решение:


      • Решить sin (β) = sin (2β) на интервале 0 ° & leq; β
        <360 °

      Я могу использовать обозначение с двумя углами в правой части, а также переставлять и упрощать; тогда я фактор:

      sin (β) = 2sin (β) cos (β)

      sin (β) — 2sin (β) cos (β) = 0

      sin (β) (1-2cos (β)) = 0

      sin (β) = 0, cos (β) = 1/2

      Синусоидальная волна (из первого триггерного уравнения) равна нулю при 0 °, 180 ° и 360 °.Но в исходном упражнении 360 ° не включены, поэтому последнее значение решения не учитывается в данном конкретном случае.

      Косинус (из второго тригонометрического уравнения) равен

      1/2 при 60 ° и, следовательно, также при 360 ° — 60 ° = 300 °. Итак, полное решение:

      β = 0 °, 60 °, 180 °, 300 °


      • Решить sin (
        x ) + cos ( x ) = 1 на интервале 0 ° & leq; x <360 °

      Хм… Я действительно ничего здесь не вижу. Было бы неплохо, если бы одно из этих триггерных выражений было возведено в квадрат …

      Хорошо, почему бы мне не возвести обе стороны в квадрат и посмотреть, что произойдет?

      (sin ( x ) + cos ( x )) 2 = (1) 2

      sin 2 ( x ) + 2sin ( x ) cos ( x ) + cos 2 ( x ) = 1

      [sin 2 ( x + cos 2 ( x )] + 2sin ( x ) cos ( x ) = 1

      1 + 2sin ( x ) cos ( x ) = 1

      2sin ( x ) cos ( x ) = 0

      sin ( x ) cos ( x ) = 0

      Ха; иди и подумай: я возведен в квадрат и получил кое-что, с чем я мог бы работать с .Хороший!

      Из последней строки выше либо синус равен нулю, либо косинус равен нулю, поэтому мое решение выглядит следующим образом:

      x = 0 °, 90 °, 180 °, 270 °

      Однако (и это важно!), Чтобы получить это решение, я возведен в квадрат, а возведение в квадрат — это «необратимый» процесс.

      (Почему? Если вы возведете что-то в квадрат, вы не сможете просто извлечь квадратный корень, чтобы вернуться к тому, с чего начали, потому что возведение в квадрат могло где-то поменять знак.)

      Итак, чтобы быть уверенным в своих результатах, мне нужно проверить свои ответы в исходном уравнении , чтобы убедиться, что я случайно не создал решения, которые на самом деле не учитываются. Подключаю обратно, вижу:

      sin (0 °) + cos (0 °) = 0 + 1 = 1

      … поэтому решение « x = 0 °» работает

      sin (90 °) + cos (90 °) = 1 + 0 = 1

      …поэтому решение « x = 90 °» тоже работает

      sin (180 °) + cos (180 °) = 0 + (–1) = –1

      … ну ладно, значит « x = 180 °» НЕ работает

      sin (270 °) + cos (270 °) = (–1) + 0 = –1

      … так что « x = 270 °» тоже не работает,

      Хорошо, что я проверил свои решения, потому что два из них на самом деле не работают.Они были созданы путем возведения в квадрат.

      Мое фактическое решение :


      Примечание: в приведенном выше примере я мог бы остановиться на этой строке:

      … и использовал тождество двойного угла для синуса, наоборот, вместо разделения 2 в предпоследней строке в моих вычислениях. Ответ был бы таким же, но мне нужно было бы учесть интервал решения:

      2sin ( x ) cos ( x ) = sin (2 x ) = 0

      Тогда 2 x = 0 °, 180 °, 360 °, 540 ° и т. Д., И разделение 2 из x даст мне x = 0 °, 90 °, 180 °, 270 °, это то же самое почти решение, что и раньше.После выполнения необходимой проверки (из-за возведения в квадрат) и отбрасывания посторонних решений мой окончательный ответ был бы таким же, как и раньше.

      Уловка возведения в квадрат в последнем примере выше встречается нечасто, но если ничего не помогает, стоит попробовать. Имейте это в виду для следующего теста.


      URL: https://www.purplemath.com/modules/solvtrig.htm

      4.2: Тригонометрические уравнения — математика LibreTexts

      Основные вопросы

      Следующие вопросы помогут нам изучить материал этого раздела. Изучив этот раздел, мы должны понять концепции, мотивированные этими вопросами, и уметь писать точные, последовательные ответы на эти вопросы.

      • Что такое тригонометрическое уравнение?
      • Что значит решить тригонометрическое уравнение?
      • Чем тригонометрическое уравнение отличается от тригонометрического тождества?

      Мы уже научились решать некоторые типы тригонометрических уравнений.В разделе 2.6 мы узнали, как использовать обратные тригонометрические функции для решения тригонометрических уравнений.

      Начало деятельности

      Вернитесь к методу из раздела 2.6, чтобы найти все решения уравнения \ (\ sin (x) = 0,4 \).

      Тригонометрические уравнения

      Когда луч света из точки \ (P \) отражается от поверхности в точке \ (R \), чтобы осветить точку \ (Q \), как показано слева на рисунке 4.1, свет образует два угла \ (\ альфа \) и \ (\ beta \) с перпендикуляром к поверхности.Угол \ (\ alpha \) называется углом падения , а угол \ (\ beta \) называется углом отражения . Закон отражения гласит, что когда свет отражается от поверхности, угол падения равен углу отражения. Что произойдет, если свет пройдет через одну среду (скажем, воздух) из точки \ (P \), отклонится в другую среду (например, воду), чтобы добраться до точки \ (Q \)? Подумайте, что произойдет, если вы посмотрите на объект в стакане с водой. См. Рисунок 4.1 справа. Снова свет образует два угла \ (\ alpha \) и \ (\ beta \) с перпендикуляром к поверхности. Угол \ (\ alpha \) называется углом падения , а угол \ (\ beta \) называется углом преломления . Если свет проходит из воздуха в воду, закон преломления гласит, что \ [\ dfrac {\ sin (\ alpha)} {\ sin (\ beta)} = \ dfrac {c_ {a}} {c_ {w}} \ ]

      Рисунок \ (\ PageIndex {1} \): Отражение и преломление.

      где \ (c_ {a} \) — скорость света в воздухе, а \ (c_ {w} \) — скорость света в воде.Отношение \ (\ dfrac {c_ {a}} {c_ {w}} \) скорости света в воздухе к скорости света в воде может быть вычислено экспериментально. На практике скорость света в каждой среде сравнивается со скоростью света в вакууме. Отношение скорости света в вакууме к скорости света в воде составляет около 1,33. Это называется показателем преломления воды. Показатель преломления воздуха очень близок к 1, поэтому отношение \ (\ dfrac {c_ {a}} {c_ {w}} \) близко к 1,33. Обычно мы можем измерить угол падения, поэтому Закон преломления может сказать нам, каков угол преломления, решив уравнение (6).

      Тригонометрические уравнения возникают в различных ситуациях, например, в Законе преломления, и в различных дисциплинах, включая физику, химию и инженерию. По мере развития тригонометрических тождеств в этой главе мы также будем использовать их для решения тригонометрических уравнений.

      Напомним, что уравнение (6) является условным уравнением , потому что оно не верно для всех допустимых значений переменной. Решение условного уравнения означает найти все значения переменных, которые делают два выражения по обе стороны от уравнения равными друг другу.

      Уравнения линейного типа

      Раздел 2.6 показал нам, как решать тригонометрические уравнения, которые сводятся к линейным уравнениям. Мы рассмотрим эту идею в нашем первом примере.

      Пример \ (\ PageIndex {1} \): (Решение уравнения линейного типа)

      Рассмотрим уравнение \ [2 \ sin (x) = 1. \]
      Мы хотим найти все значения \ (x \), которые удовлетворяют этому уравнению. Обратите внимание, что это уравнение очень похоже на линейное уравнение \ (2y = 1 \) с \ (\ sin (x) \) вместо \ (y \).Итак, это тригонометрическое уравнение линейного типа, и мы говорим, что оно линейно по \ (\ sin (x) \). Мы знаем, как решить \ (2y = 1 \), мы просто делим обе части уравнения на 2, чтобы получить \ (y = \ dfrac {1} {2} \). Мы можем применить ту же алгебраическую операцию к \ (2 \ sin (x) = 1 \), чтобы получить уравнение \ [\ sin (x) = \ dfrac {1} {2}. {- 1} (\ dfrac {1} {2}) = \ dfrac {\ pi} {6 } \).

      Напомним, однако, это не единственное решение. Первая задача — найти все решения за один полный период синусоидальной функции. Мы можем использовать интервал с \ (0 \ leq x \ leq 2 \ pi \), но мы часто используем интервал \ (- \ pi \ leq x \ leq \ pi \). В этом случае это не имеет значения, поскольку функция синуса положительна во втором квадранте. Используя \ (\ dfrac {\ pi} {6} \) в качестве опорного угла, мы видим, что \ (x = \ pi — \ dfrac {\ pi} {6} = \ dfrac {5 \ pi} {6} \ ) — другое решение этого уравнения.(Воспользуйтесь калькулятором, чтобы проверить это.)
      Теперь мы используем тот факт, что функция синуса — это период с периодом \ (2 \ pi \), чтобы написать формулы, которые можно использовать для генерации всех решений уравнения \ (2 \ грех (х) = 1 \).

      Итак, углы в первом квадранте равны \ (\ dfrac {\ pi} {6} + k (2 \ pi) \). а углы во втором квадранте равны \ (\ dfrac {5 \ pi} {6} + k (2 \ pi) \), где \ (k \) — целое число. Итак, для решений уравнения \ (2 \ sin (x) = 1 \) мы пишем \ (x = \ dfrac {\ pi} {6} + k (2 \ pi) \) или \ (x = \ dfrac {5 \ pi} {6} + k (2 \ pi) \), где \ (k \) — целое число.

      Мы всегда можем проверить наши решения, построив график обеих сторон уравнения, чтобы увидеть, где пересекаются два выражения. Рисунок 4.2 показывает, что графики \ (y = 2 \ sin (x) \) и \ (y = 1 \) на интервале \ ([- 2 \ pi, 3 \ pi] \). Мы видим, что точки пересечения этих двух кривых находятся в точном соответствии с решениями, которые мы нашли для этого уравнения.

      Упражнение \ (\ PageIndex {1} \)

      Найдите точные значения всех решений уравнения \ (4 \ cos (x) = 2 \ sqrt {2} \). Сделайте это, сначала найдя все решения за один полный период функции косинуса и

      Рисунок \ (\ PageIndex {2} \): графики \ (y = 2 \ sin (x) \) и \ (y = 1 \)

      , а затем с помощью свойства периодичности написать формулы, которые можно использовать для генерации всех решений уравнения.Нарисуйте соответствующие графики, чтобы проиллюстрировать ваши решения.

      Ответ

      Разделим обе части уравнения \ (4 \ cos (x) = 2 \ sqrt {2} \), чтобы получить \ (\ cos (x) = \ dfrac {\ sqrt {2}} {2} \). Итак, \ [x = \ dfrac {\ pi} {4} + k (2 \ pi) \] или \ [x = \ dfrac {7 \ pi} {4} + k (2 \ pi) \]
      , где \ (k \) — целое число.

      Решение уравнения с помощью обратной функции

      При решении уравнения \ (2 \ sin (x) = 1 \) мы использовали тот факт, что знаем, что \ (\ sin (\ dfrac {\ pi} {6}) = \ dfrac {1} {2 } \).{-1} (0,7) \]

      Это дает единственное решение уравнения, которое находится в интервале \ ([0, \ pi] \). Прежде чем использовать периодическое свойство, нам нужно определить другие решения уравнения в одном полном периоде функции косинуса. Мы можем использовать интервал \ ([0, 2 \ pi] \), но проще использовать интервал \ ([- \ pi, \ pi] \). Одной из причин этого является следующая так называемая «отрицательная идентичность дуги», указанная на странице 82. {- 1} (0.{-1} (0.7) + k (2 \ pi) \), где \ (k \) — некоторое целое число.

      Примечание

      Примечание. В начале упражнения для этого раздела использовалось уравнение \ (\ sin (x) = 0,4 \). Решениями этого уравнения являются
      \ (x = \ arcsin (0.4) + k (2 \ pi) \) и \ (x = (\ pi — \ arcsin (0.4)) + k (2 \ pi) \), где \ (k \) — некоторое целое число.

      Мы можем записать решения в приближенной форме как \ (x = 0,41152 + k (2 \ pi) \) и \ (x = 2,73008 + k (2 \ pi) \), где \ (k \) — целое число.

      Упражнение \ (\ PageIndex {2} \)

      1. Определите формулы, которые можно использовать для генерации всех решений уравнения \ (5 \ sin (x) = 2 \).{2} (х — 4 \ sin (x) = -3) \). Нарисуйте соответствующие графики, чтобы проиллюстрировать ваши решения.

        Ответ

        Запишем уравнение как \ (\ sin (x) — 4 \ sin (x) + 3 = 0 \) и разложим правую часть на множители, чтобы получить \ ((\ sin (x) — 3) (\ sin (x) — 1) = 0 \). Итак, мы видим, что \ (\ sin (x) — 3 = 0 \) или \ (\ sin (x) — 1 = 0 \). Однако уравнение \ (\ sin (x) — 3 = 0 \) эквивалентно \ (\ sin (x) = 3 \), и это уравнение не имеет решения. Мы пишем \ (\ sin (x) — 1 = 0 \) как \ (\ sin (x) = 1 \), и поэтому решения равны

        \ [x = \ dfrac {\ pi} {2} + 2 \ pi k \]

        где \ (k \) — целое число.

        Сводка

        В этом разделе мы изучили следующие важные концепции и идеи:

        Тригонометрическое уравнение — это условное уравнение, которое включает тригонометрические функции. Если есть возможность записать уравнение в виде

        \ (\ text {«некоторая тригонометрическая функция} x \ text {«} = \ text {число} \)

        , мы можем использовать следующую стратегию для решения уравнения:

        • Найдите все решения уравнения в пределах одного периода функции.Часто это делается с помощью свойств тригонометрической функции. Довольно часто в течение одного периода может быть два решения.
        • Используйте период функции, чтобы выразить формулы для всех решений, добавив целые числа, кратные периоду, к каждому решению, найденному на первом шаге. Например, если функция имеет период \ (2 \ pi \), а \ (x_ {1} \) и \ (x_ {2} \) — единственные два решения за полный период, тогда мы должны написать решения уравнения как \ [x = x_ {1} + k (2 \ pi), x = x_ {2} + k (2 \ pi) \], где \ (k \) — целое число:

        Иногда мы можем использовать тригонометрические тождества, чтобы переписать данное уравнение в форме уравнения (1).2 x & = \ frac {3} {4} \\ \ sin x & = \ pm \ frac {\ sqrt {3}} {2}. \ end {align} sin4x + sin2x − 1sin4x − cos4x + sin2x − cos2x − 1 (sin2x + cos2x) (sin2x − cos2x) + sin2x − cos2x − 12sin2x − 2cos2x − 12sin2x − 2 (1 − sin2x) −1⇒sin2xsinx = cos4x + cos2x = 0 = 0 = 0 = 0 = 43 = ± 23.

        Так как 0≤x≤2π, 0 \ leq x \ leq 2 \ pi, 0≤x≤2π, для sin⁡x = 32 \ sin x = \ frac {\ sqrt {3}} {2} sinx = 23 У нас есть

        x = π3,23π. (1) x = \ frac {\ pi} {3}, \ frac {2} {3} \ pi. \ qquad (1) x = 3π, 32 π. (1)

        Для sin⁡x = −32, \ sin x = — \ frac {\ sqrt {3}} {2}, sinx = −23, имеем

        х = 43π, 53π.2 х) & = 0 \\ \ Rightarrow \ cos x & = 0, ~ \ sin x = \ frac {1} {2}, — \ frac {1} {2}. \ end {align} cos2x − sin22xcos2x − 4sin2xcos2xcos2x (1−4sin2x) ⇒cosx = 0 = 0 (поскольку sin2x = 2sinxcosx) = 0 = 0, sinx = 21, −21.

        Так как 0≤x≤2π, 0 \ leq x \ leq 2 \ pi, 0≤x≤2π, для cos⁡x = 0 \ cos x = 0cosx = 0 имеем

        x = π2,32π. (1) x = \ frac {\ pi} {2}, \ frac {3} {2} \ pi. \ qquad (1) x = 2π, 23 π. (1)

        Для sin⁡x = 12, \ sin x = \ frac {1} {2}, sinx = 21 имеем

        x = π6,56π. (2) x = \ frac {\ pi} {6}, \ frac {5} {6} \ pi.\ qquad (2) x = 6π, 65 π. (2)

        Для sin⁡x = −12, \ sin x = — \ frac {1} {2}, sinx = −21, имеем

        x = 76π, 116π. (3) x = \ frac {7} {6} \ pi, \ frac {11} {6} \ pi. \ qquad (3) x = 67 π, 611 π. (3)

        Таким образом, из (1), (2) (1), (2) (1), (2) и (3) (3) (3) решения

        х = π2,32π, π6,56π, 76π, 116π.